Anda di halaman 1dari 104

Tax Outline Warren Spring 2007

1 of 104

GROSS INCOME
IN GENERAL
3 Universal Tax Policy Concerns Equity (is it fair?) Incentives (what effects will this rule have?) Administrability (will it work?) 61 Gross Income Defined All income from whatever source derived Types Salary / Fees / Commissions Fringe Benefits/similar items Interest Dividends TEST Employee benefit vs. Personal Benefit o What does it look like? Look to social norms Chalk / Air Conditioning employer Parking / Gym personal Turner Cruise Ticket Case o Taxpayer: income based on own valuation ($520) (not interested in destination, he travels more modestly than this, not worth much to him) o Govt: income based on market value of tickets ($2200) Taxpayers system is chaotic and not administrabale Court splits the difference between the two ($1400) o Luxury beyond his means / not something needed in ordinary course Issue: Individual value not always the best Old Colony Taxes paid for you represent compensation and therefore, you are taxed GROSS UP! o Figure out what to pay in order to net a certain amount Issue: Stacking (other income sources may put you at different tax rate) o Might pay tax on salary as if its the ONLY income youre making o Might pay tax on salary as if its the LAST income youre making o Might pay tax based on portion of total taxes that the salary represents

Tax Outline Warren Spring 2007

2 of 104

FRINGE BENEFITS
Something employer making you do as a part of your job 119(a)(1) Meals Requirements: o For convenience of employer 119(b)(4) if more than half the meals are furnished for the convenience of employer, then all are deemed to be o On Premises of employer (applies to employee, spouse, dependents) Rationale: o Seems less valuable to employee because it is for the employer, so we tax it @ $0 o Include wife and kids based on administrability Employer CAN discriminate based on high- and low-paid workers Kowalski State troopers being reimbursed for meals taken while working TAXED o 119 requires meals to be furnished in kind not reimbursed System is discontinuous: Reimbursement = $0 help Furnished in kind = full exclusion Congress intended to use full measure of its taxing power when creating the income tax anything that can be constitutionally taxed has been taxed 119(a)(2) Lodging Requirements: o On Premises o Employee must accept as condition of employment 119(d)(1) Campus lodging provided to employee of educational institution Qualified campus lodging provided to employee of educ institution is excluded LIMIT: 119(d)(2) o You can tax the amount of benefit based on the value of the property and the fair market rental value (what others are paying for this facility) Taxable Amount of Benefit = The lesser of: o 5% of appraised property value, or o What other, non-university officials/renters pay when renting from the University for comparable lodging

Tax Outline Warren Spring 2007

3 of 104

MINUS whatever rent the taxpayer must pay o Youll end up in the what other renters pay category if those renters pay less than 5% of the fair market value This would mean the benefit to you isnt as good in the first place (not a great deal), so we wont tax you on a supposed higher benefit NYU Rule (had people living in rent control spots where others were paying less than 5%, so why should their employees be penalized for their free lodging) Other Exclusions Rationale: o There are many things we could constitutionally tax under Kowalski, but we choose not to for some policy rationale 79(a) Life Insurance Included as income if over $50,000 + whatever employee puts in 101(a) Death Benefits 106 Contributions to Health Plans Exclude employer-provided accident and health coverage Effect: o Non-taxability of these benefits creates an incentive to provide them (overprovision may distort the system) 107 Lodging for Minister Minister of the Gospel can exclude cost of lodging without showing business premises or the convenience of the employer 112 Army No tax if in combat zone or if POW or hospitalized 117(d) Qualified Tuition Reduction o Amount of any tuition reduction to an employee is not taxable Extended to spouse and dependent children under 132(h) o ENORMOUS benefit can be anywhere o LIMIT: 117(d)(3) Must provide to all employees if you give to highly compensated employees All employees in a group of employees defined under a reasonable classification system set up which does not discriminate in favor of highly compensated If you want to give to just Professors, you need to come up with some sort of valid attributes on which to classify

Tax Outline Warren Spring 2007

4 of 104

125 Cafeteria Plans Can exclude even if participant may choose among the benefits of the plan 127 Educational Assistance Programs Amounts paid or expenses incurred by employer for education assistance are excluded Limit: o $5,250 o CANNOT Discriminate 132 (Non-Statutory Fringe Benefits) Residual way to see fringe benefits Rationale: o (Other benefits seem more obvious) Meals/Lodging cant enter market place to seek these things out (not employees choice) Health/Tuition socially optimal o Here: Administrability Efficiency/Incentives (dont want open seats on the plane) o No Additional Cost 132(b) Reqs (2): Offered for sale to customers in ordinary course of the line of business o Regular Price No substantial additional cost in providing service and no forgone revenue o Non-Substantial Loss Who counts: Employee Retired/disabled employees 132(h)(1) Spouse/dependent children 132(h)(2) Parents of employee (air transportation only) 132(h)(3) 132(i) Reciprocal Agreements Joint venture between FedEx and American Airlines is OK 132(j)(1) - Requirement of Non-Discrimination Can only give tax-exempt to highly compensated if you give to all o available on substantially the same terms to each member of a group of employees which is defined under a reasonable classification o Employee Discounts 132(c)

Tax Outline Warren Spring 2007

5 of 104

Requirements: Must be offered at cost (no lost revenue) Ordinary course of business Working Conditions Fringe 132(d) Any property or services provided to employee is excluded if it would be excluded anyway if the employee paid for it up front Gotcher Man takes trip to Japan o Dominant purpose was business and he had little control over his itinerary Excluded might have had fun, but not his purpose o 61 Economic Gain? Look to control/choice of money spent De Minimis 132(e) Any property or service the value of which is (after taking into account the frequency which similar fringes are provided) is so small as to make accounting for it unreasonable or administratively impracticable Exs: o Occasional use of copy machine o Occasional tickets o Local calls o Latenight dinner allowance might be OK as long as its only occasional and not routine/regular No non-discrimination rule Transportation 132(f) Can provide parking/subway pass Details: $100/month for commuter highway vehicle or transit pass o Must be from home to office $175/month for parking CAN discriminate Ways to get around: de minimis or working condition usually fail Gyms 132(j)(4) On-premises gym OK Reqs: Operated by employer Use is substantially by employer & family CAN discriminate We will see later, however, that the cost of operating a gym is NOT deductible to the employer as a business expense 274(a)(1) (b)

Tax Outline Warren Spring 2007

6 of 104

BUT if the gym is to be used by primarily by non-highly compensated employees, then business can deduct it Surrogate Taxation Taxing employees via taxing of the company we act like were taxing the company, but the employees are really the ones who get hurt

Ways to think about Fringe Benefits Some take Binary Approach (all or nothing) o Meal On-premise Excluded / Meal reimbursement Taxed Some Compromise o Turner / Gotcher look to purpose o Frequent Flier Miles Some exclude but with ceiling o Parking 132(f) Some exclude but with non-discrimination provision o No Additional Cost 132(j)(1) o Qualified Discount 132(c) o Qualified Education 117(d) 132 vs. Carter Admin o Carter proposed to exclude only items required as a condition of a particular job All other fringes are part of the benefit of the bargain o 132 is a residual way to see fringes Administrability Efficiency/Incentives

IMPUTED INCOME
Benefit from labor on ones own behalf OR from owning capital Housing Incentive to own your home o A makes money and rents home taxed on income o B owns home and pays not rent not taxed on rent he receives to live in his own house Counter: maybe we WANT to encourage owner-occupied living for other reasons Solution: o Give renters a special rate or deduction as an indirect means to even this distinction out (distinction exacerbated because renters generally lower income to begin with)

Tax Outline Warren Spring 2007

7 of 104

Labor Incentive for wife to stay at home o Not taxed on income she earns for work in her own home o BUT if she went to work, she would be taxed on salary and would have to pay housekeeper out of post-tax earnings Solutions: o Lower tax rate on 2nd-earners o Tax the money earned at home / Tax leisure o Tax the potential you could earn o Child care deductions to encourage you to work Problems with taxing imputed income: Administrability (Difficult to value non-market transactions) Liquidity (how to pay this tax? Cant sell your leisure) Fairness (want to encourage freedom of choice)

RECOVERY
INTRO

OF

CAPITAL

1001 Determination of Gain or Loss Gain = Amount Realized Basis o AR = money received + FMV of property received 1012 Basis Cost of Property o How much you paid for it, or o How much was included in your taxable income We may adjust based on what is logically necessary to tax you on the right amount of gain o 61 gain = income under the 16th Amdt BUT we dont tax basis You may have gained $700k when you sold the property, but you paid a price for the property at the beginning and spent income that was already taxed AR does not equal Gain Unconstitutional to tax basis o Allocation can be difficult Think of Nixons sale of inland lands

DAMAGE PAYMENTS
What were taxing when we tax damage payments is: o [What you received] [What you paid for it] o Not [What you received] [What you gave up]

Tax Outline Warren Spring 2007

8 of 104

Illustration: o Car worth $15k is hit by lightning and destroyed Insurance gives you $15k We tax AR minus the COST (what the car was worth is irrelevant)

Raytheon Settlement of antitrust suit how to tax? When you receive damage payments, we think of what they are substituting for to determine how to tax them o Test: If its replacing capital (goodwill or reputation) non-taxed If its replacing lost profits taxed Excess recovery over the amount of capital lost taxed Raytheon loses because they cannot meet burden of proof o 7491 Burden of Proof Burden is on taxpayer Shifts if taxpayer comes up with credible evidence and complies with record-keeping and produces witnesses / info / documents

HUMAN CAPITAL
Iraq Imprisonment Given $10,000 wont tax (even though $0 basis) Rationale: o Choose not to tax certain groups o Not a mkt transaction entered into freely o Compensatory o Unconstitutional Not income 803(a)(1) Nazi Statute Wont tax restitution received by victims of the Holocaust o Cash payments untaxed o Property/Assets untaxed Also, basis for any property received is the FMV of such property when received Contradiction: o We dont tax this as an involuntary transaction but we would tax an individual who had his sports car destroyed by lightning and then received a cash payment

INSURANCE
104(a)(2) Compensation for Injuries

Tax Outline Warren Spring 2007

9 of 104

GI will not include amount of damages received as compensation for personal physical injuries or physical sickness o We do not tax: Medical Expenses Pain & Suffering Compensation for lost wages (seems weird because you would have been taxed had you earned them) We leave out because it would provide weird incentives and its hard for a jury to sort out which parts are which (For non-physical injuries, we do not tax medical care) o We DO tax: Punitive Damages (For non-physical injuries, we DO tax pain & suffering and lost wages) Rationale: Involuntary Transaction

104(a)(1) Exclusion of amount received through workmens compensation acts 104(a)(3) Exclusion of amount received through accident/health insurance plans These apply for personal injuries or sickness No payment for insurance compensation if insurance is employer-provided 105(a) Amounts attributable to employer contributions Amount received from accident or health insurance for personal injuries or sickness it TAXED if the insurance is employer-provided o BUT: 103(b) Amounts received for medical care Amounts received NOT TAXED if paid to reimburse for medical care 105(c) Loss of Member Amounts received NOT TAXED if paid on flat basis ($X for a leg, $Y for an arm) and not on individual basis (Seems in the end that all we pay for is sick days) o Congress concerned about allowing a sick day program disguised as accident insurance and not collecting tax on it 106 Employer-provided Insurance Employer-provided accident or health coverage is NOT TAXED 213(a) Individual-provided Insurance You pay your own medical expenses up to 7.5% of income can deduct after that o Reality: very high bar Premium? Recovery? Incentive to provide

Tax Outline Warren Spring 2007

10 of 104

coverage? EmployerProvided Medical IndividualProvided Not taxed (106) Taxed (can deduct, but only if you meet very high bar under 213 7.5%) Not Taxed (104 or 105) BIG Incentive to provide

Not Taxed (104(a)(3))

Very little incentive to provide for self

EmployerProvided Accident IndividualProvided

Not Taxed (106) Taxed (must buy premium out of after-tax dollars with no deduction)

Not taxed (104(a)(3), 105, 105(c))

Incentive to provide

Not taxed (104(a)(3))

No incentive

Policy Concepts Surrogate Taxation o Employee may demand better health coverage because its not taxed o BUT employer may demand lower salary as a result (benefit being given to employee in tax code, but employer may act strategically) Encourages employer-provided health/accident care o The US subsidizes health/accident care if employer-provided! Small incentive for individuals to provide their own health insurance (and NO incentive to provide accident insurance o Bush: now people get lavish coverage (wasteful and hurtful to business) Large Benefit to High-Income o We base deduction on income only Two people have $1,000 in medical expenses 20% bracket = $200 benefit 50% bracket = $500 benefit o We might consider basing the deduction on a % of medical expenses Allow each to deduct 35% of medical costs 20% bracket = $350 benefit 50% bracket = $350 benefit

Tax Outline Warren Spring 2007

11 of 104

Nonphysical Injuries Business vs. Personal Distinction Dean gets slammed in newspaper for being a horrible person and has personal emotional damage NO TAX BUT if Dean gets slammed for being a horrible dean and sues for loss of business reputation TAX (business) Tort vs. Contract Distinction Burke back pay received on account of sex discrimination TAX o Tort-like rights Schleier back-pay received on account of age discrimination TAX o Not a personal injury (taxable under 104(a)(2) Reputation gets trashed after movie you were in TAX o Voluntary transaction Physical vs. Non-Physical Need physical injury it you want to exclude the money Can exclude money for emotional distress if its tied to medical care o But taxable if it goes to lost wages, pain & suffering, or punitive damages Murphy Emotional Distress Argues two things: o Statutory: Under 104(a)(2), her award should have been excluded from her gross income because it was compensation received on account of personal physical injuries or physical sickness o Constitutional: 16th Amdt taxes income gain These types of recoveries are not gain Unconstitutional Court agrees with Constitutional argument o Human capital wouldnt have been taxed if you held on to it, so cant tax the conversion of it into monetary gain o Return to in lieu of test What is the payment for? If its business reputation tax If its personal injury no tax Effects: o Under Raytheon, we wouldnt tax on emotional distress (personal), but we would tax professional reputation at least (business) o Under 104(a)(2), we would tax both o Under Murphy, we can tax neither Throws policy into a mess

Tax Outline Warren Spring 2007

12 of 104

Think of effects on conceptual soundness and administrability

ANNUITIES
Pay money in now, get certain amount back later @ regular intervals Problem: How to tax the gains? o Loan Tax actual profit each year (amount realized minus basis) o Annuity Tax average expected profit for the total contract (spreads the tax burden evenly and defers taxes)

72 Annuities Amounts received as annuities are included as gross income BUT we will NOT TAX amounts received which bear the same ratio to such amount received as the investment in the contract o This treats each annuity payment as recovery of an investment which is not meant to be taxed 3-yr annuity splits the basis in 3 ways Recognizes that part of what you are getting back in the annuity is a recovery of your own money Life Annuity Pay you a certain amount over the course of your life and divide the gains evenly depending upon your life expectancy Two situations: (assume 3yr life expectancy with $10.90 gain each year) o Die Early (after year 1) Paid in $267.30, but only got back $100 Paid taxes on $10.90 in Yr 1 OVERTAXED if you die early BUT allowed to deduct unrecovered investment 72(b)(3) o Die Later (live 10 yrs) Great Benefit $1000 Only paid taxes in first 3 years UNDERTAXED if long-life annuitant BUT we tax in full the $100 given later (exclusions cannot exceed the recovered investment) 72(b)(2)

LIFE INSURANCE
Two Types: o Pure or Term Insurance Costs up as you get older o Whole Life

Tax Outline Warren Spring 2007

13 of 104

Premiums averaged out over your life (overpay when younger and underpay when older) You get both an insurance return and a return on your capital investment Income never taxed Income on the savings account not taxed under 101

Effects: o Weve developed an industry tailored to rich people who can take money out of the taxable sector and put in this type of thing Were combining an investment account with insurance protection and giving tax benefits for it! We see complex forms of accounts as a result try to mimic a life insurance policy to get favorable treatment under the tax system Mortality Gains and Losses o Like annuities, we may have these depending on if you die early/late o With life insurance, however, you beat the system by dying early

101(a)(1) Certain Death Benefits Life insurance benefits paid because of insureds death are excluded o Means that with Whole Life, the interest return on the savings account also is excluded 79 Group-term life insurance purchased by employee Exclude cost of life insurance policy purchased by employee up to $50,000

REALIZATION
IN GENERAL
We tax events, not changes in value o Increase in property value not taxed until its sold/realized o Increase in wealth taxed when received Result: Try to structure transactions to put off events which trigger realization Rationale: o No Income youve only made money on paper o Liquidity wont have money to pay the tax o Valuation wont know value for all things

Eisner Stock Dividend given

Tax Outline Warren Spring 2007

14 of 104

Congress does not have power to tax stock dividends because there has been no realization event o No income / Gain derived from capital If he had been given a cash dividend or even had the choice to get a cash or stock dividend, that would likely be a realization event o 2 stocks now worth $110 together

Cesarini Money found in piano is taxed realization event o Congress can exert full taxing power under 16th Amdt unless specifically prohibited from doing so Haverly Receives sample textbooks and donates to library for charitable deduction Court says that either way, his net income is $0 o If he donates, receipt of books = $100, but donation = $100 deduction o If he doesnt donate, receipt of books = $0 because no realization event o (BUT he cant have #3, receipt = $0 and you still get $100 deduction) Microsoft Hypo Basis in Stock is $100 / FMV = $10M We wont tax you until you sell it But what if you want to use it? o Take out interest-free loan (bank will give it based on stock) o When you die, basis gets beefed up ( 1014) and kids can use stock to pay back loan Ways to disallow this: o Make borrowing against unrealized appreciation a realization event If you use proof of paper profits to get loan, should pay tax o Make death a realization event Not politically feasible Incentive under 1014 to hold until death because basis goes up under 1014 Benefit of Deferral Why are we so desperate to put off our realization event? Interest-free Loan $2,000 in wages, t = 40% o Tax = $800 BUT if you can defer having to pay that $800 now and instead pay in 10 years, youll pay a lot less o Equivalent of being able to borrow and pay back @ a very low rate of interest (ZERO) from the Government

Tax Outline Warren Spring 2007

15 of 104

o In the intervening years, you can invest the money and profit Present Discounted Value Non-deferred tax is $800 BUT PV of deferral $800 in 10 years @ 10% is only $308.80 o Reducing the PV of the income you receive now for tax purposes Exemption of return on after-tax deferred amount Realization doctrine is like granting an exemption on the investment income o Exemption: Yr 1 Earn $100 Taxed at some rate t (30%) o NET = $70 Yr 2 Do nothing Let that $70 earn interest $7 The $7 is TAX EXEMPT under hypo so we will net $7 in Yr 2 o Deferral: Yr 1 Earn $100 Not taxed o NET = $100 Yr 2 Do nothing Let that $100 earn interest $10 Tax that interest at some rate t (30%) o NET = $7 (Plus you pay the deferred tax of $30 in yr 2) We have $77 after Year 2 under both circumstances o Deferring the tax on an item of income is the equivalent in effect of taxing the item and then exempting from tax the income produced by investment of the after-tax proceeds

DISCOUNT OBLIGATIONS
I loan you $100 at no specified rate, but with a guarantee that youll pay me $133 in three years How to tax me?

3 Strategies: Tax me on my real-world increase in income o If interest rate is 10%, I will earn: 10 in Yr 1 (10% of 100)

Tax Outline Warren Spring 2007

16 of 104

11 in Yr 2 (10% of 110) 12 in Yr 3 (10% of 121) o PRO Reflects real world changes in value o CON No realization event (no liquidity) Defer all my taxes until the end o $0 in Yrs 1 and 2, $33 in Yr 3 o PRO Defers income undertaxes the lender o CON Bad for borrowed Divide income evenly among the 3 period (like in annuity case) o Give $11 income for each of 3 yrs o PRO Borrower happy (receive 11, but only tax on 10) o CON Accelerates Lenders income (has to tax on 11, only receive 10)

1272 Current Inclusion in Income of OID For Original Issue Discounts, we will tax an amount equal to the sum of the daily portions of the OID for each tax day during the year o We chose strategy #1 reflect real-world income Practice o Lender reports annually the amount of interest he accrues on the loan for that yr & Borrower treats an identical amount as interest he may deduct We are taxing w/o a realization event Rationale: o Certain gain from bond vs. fluctuating gain from stock o Want to reflect the real world 1273 - What OID means OID is the excess of: o Stated Redemption Price at maturity OVER issue price

INTRO

TO

CAP GAINS

We have a different tax rate for Cap Gains because of the realization requirement o Pressure to create special tax rate to encourage realization (if youre going to get nailed with big gain if you sell now because you have a low basis in the stock, then youll be less likely to realize gain low rate may lead to turnover)

Tax Outline Warren Spring 2007

17 of 104

MOTIVATION
WINDFALLS
Motivation is irrelevant to Windfalls you are TAXED o Cesarini

GIFTS
102 Gifts Gross Income does not include value of property acquired by gift o We TAX the DONOR tax his income that he uses to buy the gift o We NO TAX the DONEE dont tax A little weird because he is the one enjoying it Strategy: o Tax the one in the lower tax bracket (though not always the easiest) Irwin v. Gavitt Father gives son the income from trust for 15 years with the remainder (principal) to daughter when she reaches 21 Court: o Gift of income interest for year TAX (to son) Not a gift Want to tax SOMEONE for the income coming in currently o Prevent the transformation of income from unrealized gain If we dont have this, you could just claim that $62,000 was gifted to you and then you didnt do anything with it for 5 years and it increased to $100,000 (tax deferral) o Temporal division with realization requirement might allow for tax deferral o Gift of remainder of trust paid over 5 years NO TAX (to daughter) GIFT 102(b) Gifts of Income Income from gifts (or gifts of income from property) are TAXED o (codifies Irwin v. Gavitt) If we treat the remainder as an OID and just tax that, then the need for 102(a) disappears o Code forces an overtaxation on the recipient of the income gift o (To be economically right, tax income recipient as an annuitant and remainder recipient as an ODI)

Tax Outline Warren Spring 2007

18 of 104

273 Holders of Life or Terminable Interest Income from life or terminable interest (something with expiration date) received by gift, inheritance, or bequest is TAXED with no deduction for depreciation or PV effect of receiving income over time 1015 Basis of Gifts For gifts, donee assumes basis as if donor still had it EXCEPTION: if basis is greater than FMV at time of gift, then well use FMV at the time of gift as the basis o If B > FMV (loss has been suffered), then well use the FMV o Rationale: concern about people shifting losses within the family to the person with higher tax bracket Say you were down $2000 on a car B = $20k / FMV = $18k Incentive to gift it to higher-bracket family member and let them sell it and take the loss But 1015 curbs this if its in the red when you gift it, the FMV becomes the Basis On the other hand, say the higher-bracket family member was UP $10k on a boat B = $50k / FMV = $60k For a gift, we keep the basis at the same level as in hands of donor, so hes passing along the gain If its in the black when you gift it, the Basis stays same Testamentary Transfers 1014 Death is not a realization event At death, we get a basis step-up to FMV when property passed to kids o Old Basis = $5 o Now worth $10M Dies o Basis for inheritors = $10M How the law deals with this: Current Law o 1. Estate Tax (TAX) o 2. Step-up in Basis (no tax because death not realization event) 1/1/2010 o 1. Estate Tax repealed (no tax) o 2. 1014 - Step-up in basis repealed (TAX if die then because old basis used) o 3. 1022 goes into effect Basis for all estates bumped to $1.3M ($3M for spouse) 1/1/2011

Tax Outline Warren Spring 2007

19 of 104

o No step-up in basis Part Sale / Part Gift A buys for $30 FMV = $100, but sells for $50 o Option 1: Say he sold for 50 and gifted for 50 Gain for Dad = $20 o AR 50 minus Basis 30 Gain for Daughter when she later sells = $50 o AR 100 minus Basis 50 We still get $70 of income Dad wins because he gets to use up all of his basis o Option 2: Split the basis Gain for Dad = $35 o AR 50 minus split basis of 15 (30/2) Gain for Daughter when she later sells = $35 o AR 100 minus Basis of 65 (50 + 15) We still get $70 Seems more gift-like Reg 1.1001-1(e) (Chooses Option 1) Transferor realizes Gain to the extent that his sale price exceeds his adjusted basis But if he sells for loss, Transferor realizes no loss even if AR < Basis Problems If you give to a tax-exempt organization, well never get the gain on the later sale o Youll have low tax because we use up all of basis at time of part-sale/gift, but when the non-profit sells it later, that gain isnt taxed Solution: 1011(b) If bargain sale to charity, well only let you use up whatever portion of YOUR basis as matches the ratio of the AR to the FMV o If you sell it to a charity for of FMV, you can use of your basis (taxed on rest) If the person you give it to dies, well never get the gain on later sale because whoever gets it afterwards will benefit from stepped up basis under 1014 Business Gifts Duberstein

Tax Outline Warren Spring 2007

20 of 104

Gets Cadillac as thanks for bringing in business We look to whether this was disinterested generosity or business income o Court decides it was the later Duberstein must include as income under 61 The company, however, can take a deduction under 162 Transferee gets taxed, but not transferor

102(c) Gifts from employers Govt amends law if its an employment situation, you CANT have a 102 gift 274(b)(1) Limits on employer deductions No business deduction shall be allowed for any gift expense made directly to employee over $25 o So not only can employee not deduct, but employer cant deduct anything over $25 Goose Hypo $50 chocolate goose to employee Possibilites: o 102(c) no deduction (employee must include) o 132 could count as a fringe benefit (de minimis) BUT Reg. 1.132-6(e) Gives specific and strict examples of the type of de minimum fringe benefits allowed Employer: o No deduction if given as a gift BUT this is not a gift because of 102(c) (employee must include gift from employer) Gift = excludable under 102 + only under 102

PRIZES

AND

AWARDS

74 Prizes and Awards Gross Income includes prizes and awards BUT (b) excludes if: (2) o Received for religious, charitable, scientific, educational achievement AND o Chosen w/o entering contest o Not required to render substantial future services o MUST GIVE AWAY to government or charity Exs. Nobel Prize Money taxed if you keep it (but not if you give to govt or charity)

Tax Outline Warren Spring 2007

21 of 104

Childs trophy is taxed ($2.50) no limits Oscar goodie bags taxed as prize Oscar resort trip taxed according to Turner

SCHOLARSHIPS
117 Qualified Scholarships Gross income does not include qualified scholarships for degrees at educational organizations o Money used for tuition and related expenses o Includes tuition reduction o Does not include monies received for teaching, research that are condition for scholarship 127 Educational Assistance Programs Gross income of employee does not include amount paid by employer for first $5,250 of educational assistance program (written plan for benefit of employees) o Does not include education involving sports or grad programs Practice 4 types of students and tax treatment: o Scholarship Student Money comes in pre-tax o State Schooler Money comes in pre-tax (lower tuition via state subsidy) o Student who works at night Must pay tuition after tax on night income o Student who borrows Must pay back loan after tax on future income Rationale: o Maybe we think education is good for society But why draw line in particular place?

SOCIAL WELFARE PAYMENTS


NOT TAXED o Govt providing them in the first place (doesnt seem to make sense for government to tax and then take money back) But with that rationale, why pay federal judges out of the national treasury? (maybe we think judges will have other sources of income) 85 Unemployment compensation

Tax Outline Warren Spring 2007

22 of 104

o TAXED

COMPETING CLAIMS
DISCHARGE
OF

AND

OFFSETTING LIABILITIES

INDEBTEDNESS

Kirby Lumber Company sold bonds, value went down, so they bought some of them back o When a company issues bonds and then re-purchases them at less then the issuing price, the difference between the issuing price and the repurchase price is TAXABLE gross income (cancellation of debt) Conceptualizations Gift o Cancellation of a loan from a parent wont be taxed because it can be considered a gift from the parent o (Different in Kirby because no disinterested generosity) Purchase Price Reduction o Disagreement about price leading to compromise price doesnt mean difference between the original and the compromise will be taxed 108(a)(1)(B) Dont include if insolvent Dont include debt cancellation in income if discharge occurs when taxpayer is insolvent o If they cancel debt because youre insolvent, wont include in income o Rationale: Might not have money to pay Liquidity 108(d)(3) Insolvent Defined o Excess of liabilities over FMV of assets Ex. In Debt for 100 / have 40 in assets o Insolvent to extent of 60 If debt is cancelled, you are not taxed on 60 o TAXED on 40 108(b) Temporary Reprieve o We will reduced your basis under 108(b), so while youre not taxed on the basis now, youll end up paying tax later when you sell the assets Student Loans Looking at programs like LIPP where your loans are forgiven 108(f) Student Loan forgiveness

Tax Outline Warren Spring 2007

23 of 104

We wont tax you on forgiveness of student loan if loans is made pursuant to a program designed to encourage students to serve in occupations with unmet needs o Must work for non-profit agencies or the government Problem with LIPP: Includes for-profit jobs o Solution: Divide the Program! Non-profit side you qualify for LIPP and no tax on cancellation due to 108(f) For-profit side you qualify for LIPP but do not qualify for the tax break (must pay tax on the cancellation) o SO HLS grosses up your payment and gives you extra money to cover the tax burden because you cant cancel

Antibasis You pay tax under cancellation of indebtedness when assets are freed up Not all cancellations are taxed! o You receive money and then deduct it from income With a gain on assets, the basis occurs first (tax system keeps track of the basis) With a liability, you receive an amount first, then have to pay for it later (tax system has to keep track of amount realized) Exs. o Given $1000 which you promise to pay back. If SoL runs and you no longer have to pay, thats taxable income (frees up assets) o Hit by a bus on Mass Ave tortfeasor owes you $1000 If SoL runs before they pay, net worth returns to where it was and there is NO TAX for Cancelled Debt (received nothing and paid nothing) Need to look at whole picture from before accident ($0) to post-SoL ($0), even if he was at -$1000k at one point o Deadbeat Dad owes Child Support of $100k If SoL runs and he owes nothing (again, look at whole picture from before he had any child support obligation until after the SoL has run) Child welfare advocates dont like it; but should we use the tax code to achieve social gains? (we already do!) Zarin Atlantic City gambler arguments: o Debt never enforceable in 1st place (he didnt owe them anything) o Wasnt a cancellation, but rather a reevaluation of the price of what he received (not worth $3,435,000 only worth $500,000)

Tax Outline Warren Spring 2007

24 of 104

Tax Court: o You incurred debt of $3,435,000 and only paid back $500,000 your TAXABLE GAIN was $2,935,000 when the rest was cancelled He claimed he had no gain because Antibasis = $500k Gain = Antibasis Amount Paid Back o Court rationale: You could have redeemed those chips in for $3.4M upon receiving line of credit You received something of value in exchange for agreeing to repay the debt o Dissent (Tannenwald): In prior cases, the taxpayer had received in a prior year a nontaxable benefit clearly measurable in monetary terms which would remain untaxed if the subsequent cancellation were held to be tax free Zarin never actually received a nontaxable benefit from the loan 3rd Circuit: o Reverses the decision on grounds that the debt was non-enforceable if no debt, then cancellation is not taxed as income under 61(a)(12) o Issue: 3rd Circuit decision doesnt apply everywhere; only in 3rd Circuit in contrast, Tax Ct is national court that applies everywhere Could be taxed on full in Vegas, but $0 in AC

165(d) Gambling Losses Gambling losses are allowed as deductions only to the extent that you claim gambling gains in that same year

ILLEGAL INCOME
Collins OTB employee caught using $80k to bet on horseraces o Lost $38k / kept $42k (and subsequently repaid it) Court holds that he is taxed on the $38k o He claims he should only be taxed on the $42k o Court says that he earned chance to gamble $38k Counterarguments Tax system should stay out of the domain of criminal law Taxing the theft penalizes the victim o If thief is taxed on what he steals, he might not have enough money left to give restitution to the victim (Tax Ct has priority over victim)

Tax Outline Warren Spring 2007

25 of 104

SUM Borrowers Not Taxed Stealers TAXED o Difference? Consent of the lender (want both sides to recognize obligation to repay) James all unlawful gains taxed o Taxpayer has received income when she: acquires earnings, lawfully or unlawfully, without the consensual recognition, express or implied, of an obligation to repay and without restriction as to their disposition. Must file tax return still fact that you embezzled does not matter for selfincrimination

TAX EXPENDITURES
TAX EXEMPT INTEREST

AND THE

CONCEPT

OF

INCOME

103 Interest on State and Local Bonds Gross income does not include interest received on any state or local bond o Exceptions: Private activity bond Arbitrage Bond Effect: o Allows state and local governments to borrow at a lesser rate because the lenders dont have to pay tax on the interest they receive If you lend to a person or company and receive 10% interest, you are still subject to 35% tax rate net is 6.5% For you to receive the same amount of money from a tax-exempt bond, the state or local government only has to pay 6.5% (because tax rate = 0%) Problem: o Theres not an infinite number of 35% income people Some people are in 20% bracket, so the return to them on the 10% taxable bond is 8% SO to make that person indifferent between the taxable and nontaxable bond, the state/local govt must pay 8% interest o But this is a windfall to the rich (high-bracket) Theyll receive 8% in nontaxable, but only 6.5% in taxable

Tax Outline Warren Spring 2007

26 of 104

o The exemption of interest on state and municipal bonds is not an efficient subsidy that serves to benefit rich! Could just give direct subsidy to state/local govt Why Not? Governors/Mayors dont want it because that would make them a part of the annual budgeting process and subject to oversight; prefer to stay off the board and receive the subsidy through the backdoor Also, this way they can control the amount 103(b) Limits Tax-exemption to public bonds No exemption for private activity bonds which are not qualified bonds o If bond IS qualified tax-exempt 141(e) Qualified Bond defined Qualified Bond includes private activity bonds if it is a qualified 501(c)(3) bond 145 Qualified 501(c)(3) bond Any private activity bond issued as part of an issue if all property is owned by a 501(c)(3) organization o So the tax-exempt interest subsidy is not limited to state/local govts but can also be used by 501(c)(3)s (like Harvard) BUT 145(b) places limit on $150,000,000 of tax-exempt interest per year Universities not subject to this o Harvard saves $40,000,000 annually o Congress wouldnt give a direct appropriation of $40M to Harvard, but use tax system to accomplish through the bond market what is not politically feasible otherwise 135 Income from U.S. Bonds used to pay for college If you pay higher education expenses on redemption of bond, your income from the bond is NOT TAXED Rationale: o Incentive for people to go to college Benefit is phased out - 135(b)(2) o If modified adjusted income of taxpayer exceeds $40,000, well exclude from benefit that amount of money which bears the same ratio as the excess over $40k bears to $15k $0k to $40k = full benefit Over $55k = no exclusion What if you dont use it all for college? o 135(b)

Tax Outline Warren Spring 2007

27 of 104

If redemption exceeds college expenses, you can only exclude from income the fraction of the amount received that is spent on college If you make $30k at redemption, but only spend $10k on college, you can only exclude 1/3 of the proceeds ($20k) $6,667 The code makes you allocate some of your basis with education and some to riotous living (cant say that the original $10k your basis was all for riotous living and therefore all of the $20k proceeds should be exempt

Limits 148 Arbitrage Bond Arbitrage bonds dont get tax-exempt treatment o Any bond issued any portion of the proceeds of which are reasonably expected to acquire higher-yielding investments, or to replace funds which were used to acquire higher-yielding investments Problem: Money is fungible and hard to trace which money is for what purpose Solutions: Stacking Provision Last In, Last Out Percentage Basis some sort of allocation of bond funds collected go to higher-yielding investments Ex. o Harvard currently borrowing at 4%, but wants to put the borrowed funds in a savings account making 10% NET 6% profit BUT tax code doesnt allow you to do this sort of arbitrage

TAX EXPENDITURE BUDGET


What govt technically could be claiming but dont claim due to tax policy, various exceptions, deductions, rules, etc. o Ex. Employer-provided healthcare rules cost $493.7B

Policy arguments If we gave a subsidy/direct expenditure for healthcare, Congress would have to vote on it every year; If we use tax policy not voted on every year o Surrey liked that there was exposure via tax expenditure budget of stuff that Congress isnt voting on every year

CONCEPT

OF

INCOME

To have Tax Expenditure Budget, you must have some idealized view of income outside the tax code Haig-Simons

Tax Outline Warren Spring 2007

28 of 104

o Income is the sum of: Market value of rights exercised in consumption Change in value of the store of property rights between the beginning and end of the period [Consumption + Savings] o Would be upset that we dont tax unrealized appreciation o Haig-Simons is concerned with Uses o Current Tax Code is concerned with Sources If you want our Tax Code to equal Haig-Simons, then you should: o Close loopholes in the Code o Broaden the base o Use Consumption Tax [Y-S] = C Base C+S Who's Taxed Earners Economic Complaints Makes future consumption worth less (discourages saving) Effectively exempts income from capital Administrative Considerations Requires Basis Concept

Income Tax

Consumption Tax

Spenders

Must distinguish C from S

Effect on Saving Income Tax discourages saving Consumption tax incentivizes saving o Consumption tax is just like Wage Tax (tax wages, but not income from capital) Effects of Consumption Tax being like Wage Tax Consumption tax exempts capital o Result @ end or Yr 2 is PV as Yr 1 with wage tax This is a tax deferral and is equivalent to: o Interest-free loan o Reduction in PV of the tax o Exemption of investment income on the deferred amount All in Yr 1 100 All in Yr 2 110 Tradeoff btwn 1 and 2 50 / 55

No Tax

Tax Outline Warren Spring 2007

29 of 104

Income Tax Consumption Tax Wage Tax

50 50 [Y-S] = C [100-0] = C C taxed at 50% 50

52.50 ($5 interest taxed) 55 (no tax on interest) 55 (no tax on interest)

50 / 52.50 50 / 55

50 / 55

Wage Tax only taxes labor

DEDUCTIONS
PROFIT-SEEKING EXPENDITURES
ORDINARY
AND

NECESSARY BUSINESS EXPENSES

162 Ordinary and Necessary Business Expenses Deduction allowed for ordinary and necessary expenses incurred in carrying on any trade or business o Includes a reasonable allowance for salaries Ordinary Normal and regular part of business Gilliam o Artist goes nuts on plane wants to deduct legal fees latte NO DEDUCTION This was not an ordinary part of his business (even if he was on a business trip not ordinary Necessary Appropriate and Helpful o Doesnt have to be literally necessary Welch v. Helvering o Payments made by a commission agent to creditors of a bankrupt corporation to improve his own standing and credit are NOT Ordinary & Necessary and cannot be deducted NO DEDUCTION There was no legal obligation to pay it back not necessary 212 Individual Expenses for the Production of Income

Tax Outline Warren Spring 2007

30 of 104

For individuals, allow deduction for O&N expenses incurred for: o Production and collection of income o Management, Conservation, and Maintenance of Property held for the production of income o Determination, collection, and refund of Tax H&R Block tax lawyer to handle tax dispute (incentive for lawyer go shift all of his work into Tax billing)

Contrast: 262 Personal Exemptions No deductions ever for personal, living, or family expenses 263 Capital Expenditures No immediate deduction, but maybe later o Amount paid for new buildings or for permanent improvements to increase the value of the property Strategy: You want to cast activities as being part of some profit-making scheme so you can deduct expenses under 212 depends on facts o Stamp Collecting Hobby? No deduction Future sales venture? Deduction o Art Hanging over bed? No deduction Hanging in museum? Deduction 199 Domestic Production 9% deduction for profits from domestic production o (way to get around GATTs prohibition in export subsidies) Applies to manufacturing, production, extraction, architecture, engineering, and movies o Does NOT apply to sale of food/beverages prepared by taxpayer at a retail establishment Starbucks Hypo (advantageous to do things in central facility and sell to retailers)

SALARY
162(a)(1) Salary Businesses can deduct reasonable salaries during the year as O&N business exp Exacto Springs

Tax Outline Warren Spring 2007

31 of 104

Posner trying to make sure that company is actually paying salary and not disguising dividend payment as salary o Dividends taxed twice (as Corporate income and then ind income) o Salary deductible to company (only taxed once as ind income) Independent Investor Test o Would an independent investor approve of this bonus? Is it reasonable? Are you paying more than you should be? o [Were especially concerned when employee/owner is getting the funds] Also Father may hire son into business and pay him a disguised gift as salary This would shift income to lower tax bracket because father would be able to deduct as O&N and son would pay (gift would be treated the opposite way)

162(m) Limits on deduction of salary Can only deduct $1M in non-performance-related salary o BUT 162(m)(4)(C) Payments made solely on account of attainment of one more performance goals IS deductible [So Congress has made a political point by capping deductibility in name, but left a very large loophole to just compensate in some other way]

PUBLIC POLICY EXCEPTION


O&N expenses deductible under 162, but certain expenses are explicitly NOT deductible

162(c)(1) Bribes Cant deduct for illegal bribes or kickbacks o Even if payment is totally legal in the country where its made, we wont allow it (might put US companies at disadvantage vs. other nations who allow their companies to deduct) Rationale: o Bribor (govt) is unlikely to include it in his income and we want to tax it somehow 162(c)(2) Illegal Expenditures Cant deduct for any expenditure that is illegal under state or federal law o Ex. Cant pay to put your liquor at front of store 162(f) Fines Cant deduct fines Rationale:

Tax Outline Warren Spring 2007

32 of 104

o Undercuts the fine (decreases effective cost of fine) o Extra Deterrent Some claim, you wouldnt run the risk anyway (no benefit with fine), but this may just be a stronger incentive to avoid o Especially if fine is related to income that isnt taxable (cant ID income), then there would be an incentive to incur fines if fine was deductible 280E Illegal Business (Drugs) No deduction for expenses paid in carrying on drug trafficking operation o Applies only to drug dealers (might want to expand) o We do have 162(c)(2) as a back-up for Illegal Expenditures Legal Fees Tellier Cost of unsuccessfully defense of criminal prosecution IS deductible under 162 o Counsel fees like these are O&N biz expenses even though a suit like this may only happen once in the life of a company o TEST: Is suit connected to business or is it personal? 162(e) Lobbying Expenses No deduction allowed for communications to: o Influence legislation (exemption for local legislation) o Participate in political campaign o Attempt to influence the general public o Directly communicate with exec branch official Effect: o Helps consumers by diminishing the amount businesses would be willing to spend on lobbying Reduces the disparity in economic power btwn groups

CAPITAL EXPENDITURES
Capital Expense 263 Personal Expense 262

O&N Business Expense 162

263 - Capitalization Requirement No immediate deduction will be allowed for new buildings or for permanent improvements made to increase the value of any property or estate

Tax Outline Warren Spring 2007

33 of 104

o Capital Exp capitalize over life under 263 o O&N deduct immediately under 162 o Illustration Buy Greenacre for $1000 in Yr 1 We will not let you deduct that $1000 immediately We want to account for changes in net worth (havent lost $1000 because youve received an asset) Woodward Litigation/Acquisition costs associated with the appraisal of stock must be capitalized because whatever you paid for the stock would have been capitalized o Why capitalize the stock? No decrease in net worth Benefit carries forward past this first year INDOPCO Debate over fees associated with a friendly takeover If the benefits of expense will extend beyond the year in question, you must capitalize even if net worth has not gone up/down o Problem: EVERYTHING has some sort of future benefit potentially Goodwill/relationships/etc. Applications post-INDOPCO Repair vs. Improvement o Repair Deduct o Improvement NO o Rule 2001-4 Original Use Deduct / New Use NO Original Life Deduct / Life Extension NO [Leads to very complicated factual inquiries] o i.e. what did they do to the plane? Contract Rights o $300 Fire Insurance Premium for small business Capitalize over life of policy NO Deduction Self-constructed Asset o Buy Building Capitalize o Build own office building may be able to classify part of costs as salary and part as materials

Tax Outline Warren Spring 2007

34 of 104

Salary o Do not have to capitalize salary, even if technically employee is thinking of ideas in current year to benefit you in future

Advertising o Advertising Currently deductible (even though probably designed to produce business in future) o Product Development capitalize (future benefit) Incentive to characterize as Advertising New Business/Function vs. Existing Business/Function o If its something new that you do capitalized o If its an ongoing/regular thing O&N We want to ensure proper classification because we dont want to give existing businesses an unfair advantage over startups Ex. o Fidelity has mutual funds and wants to create a new investment vehicle in environmentally conscious companies o Need to decide what it is that Fidelity does and what this new project is

SUMMARY of Post-INDOPCO world Business community goes nuts after INDOPCO because now everything could potentially be capitalized because everything could have a future benefit past this year o Treasury Reg. 1.263(a)-4(e)(4)(j) The following are treated as costs that do not facilitate acquisition: Employee Compensation Overhead De Minimis Costs (anything inside your company is going to be able to be expensed) o Rationale: Hard to divide up and distinguish some of these internal costs Simplify the code at the expense of being accurate

Why this is worth fighting about? If you are able to expense immediately, you are DEFERRING Tax (paying less in the present moment by taking something OUT of your income) o We want to accelerate a deduction to make it more worthwhile o PV of Tax paid now > PV of same amount of tax paid in the future

Tax Outline Warren Spring 2007

35 of 104

By allowing immediate deduction of capital expenditures, you are reducing the effective tax rate on the expenditure to 0% o Capitalized Expenditure Pay $100 for Greenacre Sell 5 yrs later for $200 $100 return at 30% tax = $70 profit 100% pre-tax profit 70% after-tax profit o Immediately Deductible Capital Cost Pay $100 for Greenacre but take immediate deduction Saves you $30 in Yr 1 ($100 less of taxable income) After-tax cost is just $70 (paid $100 but $30 deduction) Sell 5 yrs later for $200 (but with $0 basis its been expensed) Tax is now $60, so after-tax gain at sale is $140 Cost was $70 Gain was $140, so AFTER-TAX PROFIT was 100% Same as PRE-TAX Profit when Capitalized above The After-Tax Profit is 100% when we deduct immediately Some industries have been singled out under 174 or 263 for immediately expensing (0% effective rate of taxation) o Newspaper Circulation Expenses o Research & Experimental Expenditures o Soil & H20 Conservation Expenditures o 263A(e)(4) Farming Business Exempts Christmas Trees o 263A(h) Creative Expenses Dont have to capitalize any qualified creative expense Literary manuscript, musical composition, etc. Income vs. Consumption Tax o Move Expenses Forward (Deduct Immediately) Income-Tax friendly o Defer Expenses (Capitalize) Consumption-Tax Friendly

CAPITAL RECOVERY PROVISIONS


Having accepted that we have to capitalize rather than currently deduct, when exactly can you deduct these capitalized expenses?

LOSSES
Remember that losses may be acceptable in some cases as offsets to taxable income

Tax Outline Warren Spring 2007

36 of 104

165 Losses Can deduct for any loss sustained which is not compensate for by insurance o 165(c)(1) losses incurred in trade or business o 165(c)(2) losses incurred in profit-seeking activity How much can you deduct? o 165(b) You can deduct for losses up to the amount of your basis Limits: Business or Investment Property NOT Personal Property BUT losses are not limited to investment gain; can deduct losses from salary income, interest income, etc. Requirement of a Realization Event Cottage Savings o Loss realized only when the properties exchanged are materially different Taxpayer tries to argue that the two sets of mortgages entail different rights/obligations Commissioner looking for change in economic position Court rules for TAXPAYER based on the change in legal obligation framework Who does this benefit? o In the present instance, we have a LOSS IRS wants to make it hard to realize the loss (since its a deduction from income) o BUT in most circumstances, we have a GAIN IRS will benefit in long run if gains are easy to realize 165(c) Personal Assets Trying to figure out if we could deduct a loss on personal home NO o Losses limited to those: Incurred by trade/business Incurred in profit-seeking transaction, or Fire/storm/shipwreck o BUT if the house went UP in value, youd be taxed on the gain Rationale: Maybe we dont want you to be able to deduct for general wear-and-tear because were already helping you out with imputed income 165(d) Gambling Losses Gambling losses are only deductible to the extent of gambling winnings o If you just go and lose, youre out of luck 1211(b) Capital Losses

Tax Outline Warren Spring 2007

37 of 104

Youre limited on deduction of capital losses to the extent of your capital gains for that year + an extra $3000 allowance (much like gambling) Rationale: o You control the realization event (concerned that youll trigger the loss to offset a salary gain) We wont let you pull out just the losing stocks in your portfolio Stock A is up $70k dont realize gain Stock B is down $70k rush to realize loss to offset salary Effect: o Sure, it hurts the manipulators o BUT it also hurts the true losers (the guy whose only stock is losing and he needs to get out can only deduct up to $3,000/yr) Solution: o Maybe we should disallow a deduction if you have other stocks which have gone up in price, even if you havent sold/realized gains (some administrability problems)

267(a) Related Parties No deduction shall be allowed in any LOSS from the sale or exchange of property, directly or indirectly, of related persons (or other specified relationships in 267(b)) Partial reprieve for later Gain: o If the original sale was at a loss and the new owner later sells it at a gain, the earlier loss may be used to offset the gain for tax purposes (no similar treatment, however, if later sold for another loss)

Father's Basis Sold to child for DISALLOWED LOSS Child's resale price "Realized"Gain or Loss Total Family Gain or Loss Recognized Gain or Loss for Child

Later sold for Later sold Gain for Loss 1000 1000 800 800 (200) 3000 2200 2000 2000 (200) 500 (300) (500) (300)

Child able to offset his fathers $200 loss (which was earlier disallowed) against his own $2200 gain Total of $2000 gain

Tax Outline Warren Spring 2007

38 of 104

Child not able to use his fathers $200 loss (which was early disallowed) on top of his own $300 loss Total remains $300 loss even though family lost $500 as a unit o Rationale: Congress is more concerned about manipulation of losses than gains Gain is based on FAMILY Gain Loss is based on PERSONAL Loss

1091 Wash Sales If taxpayer sells or exchanges stock or securities at a loss and then regains substantially identical stock within 30 days, no loss deduction is allowed o Prevents you from selling stock for loss on December 30 and then rebuying it 2 days later Selling for Gain: o TAXED on the Capital Gain even though you rebuy You must realize gain! But why would you want to? (2) If youre in a low tax year, you might want to realize gain in that year If youre in a high tax year, we dont allow you to wash the loss (would prefer to deduct from income in high-tax year) May want to have capital gains this year to offset capital losses under 1211(b) Remember, under 1211(b), you can only deduct for capital losses to the extent you have enjoyed cap gains (+ $3k) Partial Reprieve: o 1091(d) Unadjusted Basis in case of Wash Sale When rebuying stock, the basis shall be the basis of the stock sold or disposed of, increased or decreased by the difference between: The price at which the property was re-acquired AND The price at which the substantially identical stock or securities was disposed of Basis2nd = PriceBought2nd PriceSold1st + Basis1st This basis applies whether you rebought at loss or gain Rationale for adjusting the basis: If you sold it @ $750 and rebought at $1100, there was a $350 gain that happened while you werent in market and Congress says you shouldnt be taxed on that (via a smaller basis) Take your original basis and add $350 to it

Tax Outline Warren Spring 2007

39 of 104

Likewise, if the stock went down $150 while you were out, your basis also goes down you shouldnt be able to take advantage of that (via an enlarged basis) Take your original basis and subtract $150 from it Repurchase at Gain 1000 750 (250) 1100 Repurchase at Loss 1000 750 (250) 600

1091 Basis Sold For (dump at years end) DISALLOWED LOSS Wash Purchase (repurchase within 30 days) How you might think of basis

How 1091 calculates basis

1100 + 250 = 1350 1100 750 +1000 = 1350

600 + 250 = 850 600 750 + 1000 = 850

Fender Trusts sell bonds to bank on 12/26 (at a loss) and repurchase from bank at nearly the same price on 2/6 o 1091 (wash sale) inapplicable waited 31 days o 267 (loss due to sale to related party) inapplicable Fender only controlled 40.7% of the bank (short of 50%) Emphasizes substance over form o NOT a bona fide loss (hasnt really disposed of the asset) o Exam Strategy 1091 (wash sale) 267 (related parties) FORM (no real economic loss) Sale of identical securities How to deal with situation where taxpayer sells share of stock, but previously owned 3 shares all with different basis o Need to figure out which stock was sold because stocks are fungible o Solution: Court uses FIFO unless taxpayer can recognize that hes selling a specific share o Other proposals: LIFO Force them to use highest basis Force them to use lowest basis

Tax Outline Warren Spring 2007

40 of 104

Averaging (Clinton administration preference)

BAD DEBTS
If you loan $1k to a party who defaults, youve lost $1k in H-S world o Want to deduct that

Generes Taxpayer makes $300k loan to business where he is 44% owner; company goes bust and he wants to deduct the loan as a bad debt business loss Court looks to the Dominant Motivation of the taxpayer o Was his motivation to save the company or to save his investment? Ct finds motivation was save investment that is not a legitimate business reason, so he cannot take the deduction 166 Bad Debts Deduction allowed for any debt that becomes worthless 166(d) rule does only applies to business debts o Debt created or acquired in connection with a trade or business of the taxpayer How rule applies to nonbusiness profit-seeking activities and personal debts: o Profit-seeking 166(d) wont allow Bad Debt deduction Just deduct it like a capital loss now BUT limited under 1211(b) can only deduct capital losses to the extent of your capital gains + $3k allowance o Results in overtaxation of this loss (because you cant deduct in full) o Personal 166(d) wont allow Bad Debt deduction But you can deduct as a capital loss subject to 1211(b) Can only deduct capital losses to the extent of your capital gains + $3k allowance o Since this deduction is a benefit not previously afforded to individuals, this results in undertaxation of this loss o Overall Effect: We have expanded deductability for personal losses, but contracted deductability for profit-seeking activities Puts a premium on classifying the bad debt loss as a business loss rather than just profit-seeking

Tax Outline Warren Spring 2007

41 of 104

Business 162 Allow deduction for O&N

Profit-Seeking

Personal

Expenses

212 262 Allow deduction for No Deduction production and collection of income, etc. 165(c)(2) Deduct for losses incurred in profitseeking activity 166(d) No deduction for nonbusiness debts BUT you can deduct like a capital loss subject to the limits of 1211(b) 165(c)(3) No deduction for personal losses unless casualty 166(d) No deduction for nonbusiness debts BUT you can deduct like a capital loss subject to the limits of 1211(b)

Losses

165(c)(1) Deduct for losses incurred in trade or business 166(a) Deduct for bad business debts

Bad Debts

Bad Debt vs. Gift Want to look to substance of transaction o Bad Debt Expectation of getting repaid Deductible But TAXED to defaulting party as Cancellation of Indebtedness o Gift No repayment expectation Non-deductible to giver But deductible to party that receives it

DEPRECIATION
167 Depreciation Reasonable allowance for the exhaustion and wear & tear of property used in a trade or business ( 167(a)(1)) or for the production of income ( 167(a)(2)) Theory Start with Basis under 1011 Depreciation is a way of recovering that Value of asset is declining over time o Makes sense that we would reflect that change in value/change in wealth Mirror Image of OID were certain value will decline, so we allow deduction What assets count?

Tax Outline Warren Spring 2007

42 of 104

Wasting Assets those with limited lifetime o Machine YES o Land NO

197 Amortization of Goodwill and other intangibles Intangible assets may be amortized over 15 years o Arbitrary number If your shelf life is less than 15 years, you want to stay out of this intangible asset category because you could deduct more of your basis early on with a calculated life of less than 15 years If your life is more than 15 years, you would want to squeeze into this compromise number Ex. o Goodwill reputation/legitimacy Why you pay $250 for a newspaper company even if the printing press and subscribe list are worth $100 each $50 Goodwill o License Permits o Patents/Copyrights o Customer-based intangibles o Non-compete covenants (Complete list at 197(d)(1)) Sports teams explicitly left out under 197(e)(6) 197(c)(2) Self-created intangibles not included o Advertising! If you go out and buy goodwill amortize If you go out and create it via advertising Currently deductible! World Publishing Taxpayer buys land subject to the continuing lease of a building constructed by tenant on the land; purchase price includes both the land and the building, though parties stipulate that the buildings useful life would expire upon expiration of the lease o FMV of land is $400k taxpayer spent $700k o Now wants to deduct for depreciation of building Arguments: o TP: (Deduction) I spent $700k for a reason; $300k on something! o Govt: (No deduction!) Taxpayers never going to be able to use the building because useful life < lease term Lessee is already deducting for depreciation Held: o Court holds that taxpayer may take a deduction for depreciation of the building

Tax Outline Warren Spring 2007

43 of 104

Both parties have a separate wasting investment in the property (Blackmun undisturbed that parties were simultaneously depreciating the asset) o Alternative argument from Warren: Taxpayer could have amortized the Premium Lease Rent was locked in at pre-Depression level, so maybe the extra $300k was spent on that the Premium (above-mkt) Lease has a limited lifetime and may properly be seen as a depreciable asset So we might conceptualize a depreciable asset where it wasnt wholly apparent at first (Blackmun mentions this, but doesnt rely on it)

CALCULATING

THE

ANNUAL DEPRECIATION DEDUCTION

Economic Depreciation Expected decline in real-world value of a wasting asset Exs. Chalk o Produces steady income stream of $1000/yr for 3 yrs (paid on 12/31) o On 1/1 of Yr 1, the chalk is worth $2,486 PV of Yr 1 payment = $909 PV of Yr 2 payment = $826 PV of Yr 3 payment = $751 = $2,486 o At the start of Yr 2, the chalk is worth only $1,735 PV of Yr 2 payment = $909 PV of Yr 3 payment = $826 = $1735 The chalk has depreciated from Yr 1 to Yr 2 by $751 o At the start of Yr 3, the chalk is worth only $909 PV of the Yr 3 payment at that time is $909 The chalk has depreciated from $1735 to $909 = $826 o At the start of Yr 4, the chalk is worthless No more useful life The chalk has depreciated from $909 to $0 = $909 o With a steady income stream, we will see decelerated depreciation where the opening years have low depreciation and later years have more coincides with constant income stream Baseball Player or Machine o Typical player who produces a little less with age Yr 1 = 1300, Yr 2 = 1200, Yr 3 = 1100 $3600 o On 1/1 of Yr 1, the player is worth $3000 PV of Yr 1 income (1300) = 1182

Tax Outline Warren Spring 2007

44 of 104

PV of Yr 2 income (1200) = 992 PV of Yr 3 income (1100) = 826 = 3000 o At start of Yr 2, the player is worth $2000 PV of Yr 2 income (1200) = 1091 PV of Yr 3 income (1100) = 909 = 2000 The player has depreciated by 1000 (3000 2000) o At the start of Yr 3, the player is worth $1000 PV of Yr 3 income (1100) = 1000 The player has depreciated by 1000 (2000 1000) o At the start of Yr 4, the player is worthless No more useful life The player has depreciated by 10000 (1000 0) o This pattern of $1000 depreciation each year is straight-line depreciation coincides with declining income stream Different Baseball Player o This player breaks down faster Yr 1 = 1000, Yr 2 = 500, Yr 3 = 250 $1750 o On 1/1 of Yr 1, the player is worth $1510 PV of Yr 1 income (1000) = 909 PV of Yr 2 income (500) = 413 PV of Yr 3 income (250) = 188 = 1510 o At the start of Yr 2, the player is worth $661 PV of Yr 2 income (500) = 455 PV of Yr 3 income (250) = 206 = 661 The player has depreciated by $849 (1510 661) o At the start of Yr 3, the player is worth $228 PV of Yr 3 income (250) = 228 The played has depreciated by $433 (661 228) o At the start of Yr 4, the player is worthless No useful life The played has depreciated his final $228 o This pattern of highest depreciation in the early years is known as accelerated depreciation coincides with an exponential decline in income each year (not a mere drop)

How to deal with these different types of assets? The code might try to simplify and just do straight line depreciation (taking a set portion (100/yrs%) of the value each year) But we see here that this wouldnt necessarily be fair for all assets o Baseball Player A wouldnt mind because he would be taxed just right (straight-line would reflect the steady decline in value over the years) o BUT:

Tax Outline Warren Spring 2007

45 of 104

For the Chalk, straight-line would undertax in the early years and overtax in later years Able to deduct/depreciate more in early years than they are really losing in real-world value (system would be saying that they lost $850, but depreciation was only $751) For Baseball Player B, straight-line would overtax in the early years and undertax in the later years Not allowing enough deduction the early years to reflect their real-world loss in value Our depreciation system should reflect whatever we think the real-world looks like in terms of how assets lose value o If we think that assets lose value exponentially at the beginning and then slow at the end of their useful life, then we would want to give everyone accelerated depreciation, allowing them to deduct a lot early on But if we do this accelerated depreciation and the real-world is controlled loss of value (like with the typical baseball player in Ex 2), then were back to the problem of undertaxing early on (by allowing deductions for more than the real-world loss of value) and then undertaxing in the end

Tax Depreciation Recovery Period o Time over which asset is wasting You prefer shorter because can deduct earlier on o 168(c) Applicable Recovery Period o 168(i) Class Life Make an attempt in 1986 to try to assess how long things last in the real world o 168(e) Classification of Property (quickly get away from the 168(i) scheme) We put things in categories and effectively discount the length of their useful life i.e. if class life is 9 years, we treat it as 5 yrs under 168(e) Effect: By discounting the number of years in the useful life, we are effectively lowering the tax able to deduct a greater percentage of the value in earlier years than youve lost in a real-world sense o Congress has toyed with the numbers for certain industries and lowerd the useful life of certain assets Provides a tax advantage without as much political accountability Applicable Convention

Tax Outline Warren Spring 2007

46 of 104

o 168(d) - Half-year Convention if you buy within the year, we assume -year possession and depreciation credit Depreciation Method how are we doing the deductions: o Three Kinds: 168(b)(1) 200% Declining Balance (Double Declining) Use: (Default standard) unless noted otherwise Method: Find the useful life results in a percentage that would be deducted using straight-line (5 yr life would be 20%/yr) Double that percentage and take that out every year (40% of basis, then 40% of remainder, etc) Then, under 168(b)(1)(B), we switch to straightline in the 1st year when using straight-line will yield a larger allowance (If we didnt do this, wed go forever) Ex.: 5 year-asset bought for $1000 Yr 1 Normally would deduct 40% ($400), but we have the half-year convention $200 Yr 2 remainder is $800, take 40% = $320 Yr 3 remainder is $480, take 40% = $192 Yr 4 remainder is $288, take 40% = $115.20 Yr 5 - 168(b)(1)(B) if we kept up with DDB, wed deduct 40% of the remaining $172.80 = $69.12 BUT if we did Straight-Line at this point, wed have $172.80 divided by the last 1.5 years of useful life = $115.20 (switch!) Yr 6 now just use the final $57.60 for the last yr 168(b)(2) 150% Declining Balance Use: 15-20yr property Farming Business Method: Find the useful life results in a percentage that would be deducted using straight-line (5 yr life would be 20%/yr) Multiply that percentage by 1.5% that percentage and take that out every year (30% of basis, then 30% of remainder, etc)

Tax Outline Warren Spring 2007

47 of 104

Then, under 168(b)(1)(B), we switch to straightline in the 1st year when using straight-line will yield a larger allowance (If we didnt do this, wed go forever) Ex.: 5 year-asset bought for $1000 Yr 1 Normally would deduct 30% ($300), but we have the half-year convention $150 Yr 2 remainder is $850, take 30% = $255 Yr 3 remainder is $595, take 30% = $178.50 Yr 4 168(b)(1)(B) if we kept up with DDB, wed deduct 30% of the remaining $416.50 = $124.95 BUT if we did Straight-Line at this point, wed have $416.50 divided by the last 2.5 years of useful life = $166.60 (switch!) Yr 5 now that weve switched, we stick with it $166.60 again Yr 6 now just use the final $83.30 for the last yr Always want to test the 168(b)(1)(B) point! 168(b)(3) Straight-Line Method Use: Nonresidential real property Residential rental property (and others expressly mentioned) Method: Divide total basis by # of years Ex.: 5-year asset bought for $1000 Yr 1 normally would be $200, but were on year convention, so only $100 Yr 2 - $200 Yr 3 - $200 Yr 4 - $200 Yr 5 - $200 Yr 6 - $100 With year convention, we spread over 6 taxable periods

Policy Implications Like with applicable recovery periods, Congress has toyed with the methods for certain industries and effectively changed the tax rates without much political accountability o Congress can put people anywhere it wants on the spectrum of: 0% immediate expensing of a capital investment 35% economic depreciation according to your nominal tax rate

Tax Outline Warren Spring 2007

48 of 104

o Ex. Want to spur development in NYC Post-9/11 3 ways Congress can do it: Shorter recovery period Use of DDB Give bonus depreciation in the 1st yr Investment Tax Credits aim to get effective tax rate to 0% $1000 investment, t = 48% $480 Give 10% credit = $100 Still need to make up $380 Look to PV of tax benefits over 5 yrs using applicable depreciation method But Tax Credits are preferable because theyre more visible

Sale of a Depreciable Asset 1016(a)(2) Adjustments to Basis Every dollar of depreciation deduction reduces the adjusted basis by $1 o Future tax on the gain or loss from the sale takes into account previous deductions BUT the benefit is that you are deferring the tax (able to pay less in tax now because of depreciation, but youll just pay more later when you sell because of lower basis) o Ex. 5 yr asset purchased for $1k with Straight-Line Depreciation Yr 1 deduct $200 ($800 basis) Yr 2 deduct $200 ($600 basis) Yr 3 deduct $200 ($400 basis) Then SALE for $800 Taxed on $400 gain Math adds up though: you bought for $1000 and sold for $800, so you really had a $200 loss over the long-haul But thats equivalent to a $600 deduction + $400 gain o Problem: The gains from the subsequent sale ($400) will likely be taxed at a lower capital gains rate than the ordinary income deductions Deduct from high-tax rate TAXED on a low-tax rate o System must develop way to RECAPTURE Transferability of Tax Benefits & Corporate Shelters

Tax Outline Warren Spring 2007

49 of 104

In form, the benefits apply only to the owner, but contractual agreements can be constructed to transfer the depreciation benefits

Exs. Airplanes o Airplanes owners have potential for big depreciation deductions but need to have income from which to deduct o So American Airlines partners with someone with income CitiBank Citibank reaps tax benefit via deduction American Airlines leases plane back from CitiBank at very low cost (also benefit) AA owns the plane in every respect but the legal/tax sense Desks in college classroom o HLS doesnt need depreciation deduction from desks/lights in classroom because theyre not paying taxes anyway! o Kirkland & Ellis should own the desks and rent them at low-cost to HLS HLS gets low-cost equipment K&E gets depreciation deduction 168(g) Alternative Depreciation System for Certain Property (a bit of a buzzkill for above arrangements) For property that is used predominantly outside the US or is used for tax-exempt purposes: o Straight-Line Depreciation must be used o Useful Life is extended Effect: o Benefit is still available, but its not worth as much TIFD If you use up all of your depreciation, come up with some transaction o How they did it: Created Partnership GE brings in $590M (plane, accts receivable, cash) Dutch banks bring in $117.5M o 82% vs. 18% Split Operating Income from the Partnership GE gets 2% Dutch Bank gets 98% o Since the depreciation has run out under US law, the Dutch income should be taxable o BUT they are a foreign corporation, so no tax on them o Also, they get 80% depreciation deduction under their own law

Tax Outline Warren Spring 2007

50 of 104

Final Cash Distribution Dutch Bank gets original investment + 9% RoR GE gets everything else Court rules that the Dutch banks which held partnership interests in the Castle Harbour partnership were not bona fide equity partners o Pass-thru entity / no meaningful stake in the success of failure of the company Use Totality of Circumstances to determine if they were really partners economic realities matter, not artificial constructs Tax Code requires that partnerships allocate tax liability based on the economic realities of the partnership o Look at the whole picture / overall effect Courts/Congress always wary when you try to shift income to a tax-insensitve unit

DEPLETION
Cost Depletion Estimates total amount of natural resource in property and allows a deduction of its cost in proportion to the years extractions 613(a) Percentage Depletion For oil wells, mines, and other natural deposits, percentage depletion is allowed o Allows deduction of a specified % of gross income based on estimates of typical recovery (without regard to cost) o Can deduct forever REMAINS deductible even after the basis is recovered o BUT if you buy a real asset like drilling equipment you can only deduct up to the cost of full basis How this works Real-world o Pay $5 for mine o Recover $40/yr for 3 yrs o Sell mine for $5 Earn $120 total ($5 + $120 - $5) % Depletion o Earn $40/yr o Deduct 20% ($8) under 613 Taxable income for 3 years is $32 + $5 for selling mine = $101 Total deduction for 3 years is $24 ($19 above what you paid) We are giving a subsidy to oil companies via our depletion basis rules! o Drillers get to deduct way more than their investment costs

Tax Outline Warren Spring 2007

51 of 104

o Two solutions: Make you max out % depletion @ $0 (no matter what you sell, cant deduct) Modify your basis at the time of sale of the mine by making your basis go negative Other relevant provisions 263(c) Capital Expenditures (Intangible Drilling Costs) Intangible drilling costs for oil and gas wells can be expensed rather than capitalized 616 Development Expenditures Allows as a deduction expenses for the development of a mine or other natural resources (other than oil or gas wells) if those costs are incurred after the existence of ores/minerals has been found o Doesnt apply to expenditures for the acquisition or improvement of property subject to depreciation rules under 167

BUSINESS/PERSONAL BORDERLINE
IN GENERAL
Started with business stuff + profit-seeking expenditures Now: Border o If its 262, its never deductible

Clothing Pevsner If its adaptable to be worn elsewhere, you cant deduct as a business expense o She tried to argue that she wouldnt wear these clothes and should get to deduct the expense as a result, but the court rejects this subjective test o Rationale: Administrability court cant be looking at every little thing o Contrast: Ballroom dancer could deduct her shoes Childcare Argue that childcare is a business expense because you couldnt work without it 21 Childcare Tax Credit Tax Credit of some % of expenses up to $6,000 o [Important that its a Credit rather than a Deduction] Credit is same $$$ amount no matter what tax-bracket Deductions are worth more to higher-bracket people (able to deduct certain amount from income and save more when taxed)

Tax Outline Warren Spring 2007

52 of 104

Phase Out: o Reduced by 1% for every $2000 that your AGI exceeds $15k $15k 35% of $6000 = $2100 $25k 30% $35k 25% o Reduction grounded at 20% No matter how much you make, you can always deduct 20% of $6,000 or $1200 (Credit is regressive up until $43,001, then flat @ 20%) Effects: o High-income people still get some benefit o Lowest income people dont really take advantage because income is so low that they wont pay tax anyway o Might encourage people to enter the marketplace (overcome disincentive of 2nd-earners going to work) Whos covered? o 21(b)(2)(A) The following employment-related expenses if such expenses are incurred to enable the taxpayer to be gainfully employed Expenses for household services Expenses for the care of a qualifying individual: o Dependent under the age of 13 o Dependent who is physically or mentally incapable o Spouse who is physically or mentally incapable o Might a butler fit in? Depends on how we construe the regs Reg. 1.44A-1(c)(2) must be attributable in part to care of the dependent

129 Dependent Care Assistance Programs Can exclude up to $5,000 by employer to for dependent care assistance o Cant exclude more than the amount your spouse makes unless your spouse is a student or incapable of working (the reason being that we wont give you dependent care assistance if you could do it yourself for less) Employer cannot discriminate wrt this program

TRAVEL

AND

MOVING EXPENSES

162(a)(2) Travel and Business Expenses Allowed a deduction for travel expenses (including non-lavish meals and lodging) while: o Away from home o In the pursuit of a trade or business

Tax Outline Warren Spring 2007

53 of 104

Home Commuter Cost from Suburbs Not Deductible o We consider it your choice to live in suburbs (contrast with childcare and so-called choice to have kids) Commuter Cost from NYC to Boston (live here Monday AM to Friday PM) o Technically youve met the definition under 162(a)(2) o BUT Commissioner doesnt like it: Home means Place of Business Since you work in Boston, youre not away from home when youre here And on the weekends in NY, those expenses are not deductible because theyre personal McCabe o NYC police officer needs to carry a gun; most direct route to job is through NJ, but that would require a permit wants deduction for extra cost of commute NO (might feel differently if he were a nuclear bomb technician and was not allowed to live within 50 miles) Away From Correll o away from means overnight Cost of meals is deductible only if the trip requires taxpayer to stop for sleep or rest need something to put you in travel status (Boston lawyer in NYC for one-day trial = NO DEDUCTION for meals) o (Travel can be deducted as O&N whether its flight to NYC or cab in Boston) Hautzis o Tries to deduct expenses for summer clerkship in NYC o Arguments: Taxpayer: away from home (Boston) in pursuit of business (summer job) Commissioner Home = place of business o Working in NY, so that was home o Court: She was no business connection to Boston, so shes not away from home o (Lets face it: this seems pretty personal) Congressmen - 162 o District = home

Tax Outline Warren Spring 2007

54 of 104

o May deduct up to $3k for travel/living expenses in DC 274(n) Disallowance of certain entertainment expenses Congress halves everything! only 50% of meals and entertainment expenses are deductible o Whoever pays for meal gets the deduction Rationale: o is personal benefit o is for employer use Assumption is that 50% of meal is personal consumption and the other half is business-related Moving Expenses 82 Reimbursement of Moving Expenses Except as provided in 132(a)(6), moving reimbursement is INCLUDED in income 132(a)(6) Qualified Moving Expense Reimbursement Fringe Gross income shall not include qualified moving expense reimbursement o Defined under 132(g) Qualified Moving Expense Reimbursement Amount recovered by an individual from employer as reimbursement which would be deductible if individual paid for it himself under 217 217 Moving Expenses Deduction shall be allowed for reasonable expenses of moving personal goods and traveling if: o Move more than 50 miles o Work a job for a certain amount of time 39 weeks at employer in next yr 78 weeks at self-employment in next 2 yrs ALL moving expenses are deductible under 217 provided you move 50 miles and have some sort of job on the other end o Doesnt matter who pays (employer reimbursement or you personally) General comment about reimbursements Employee has travel expenses of $100 that are reimbursed by the firm 2 choices: o He can ignore it because its reimbursed Must trust the taxpayer o He can report it but then take it as an O&N expense under 162 (a)(2) Must save a ton of receipts complicated

Tax Outline Warren Spring 2007

55 of 104

IRS: o You can ignore the expense provided the employer acknowledges that they are complying with the tax law

ENTERTAINMENT
274 Disallowance of Certain Entertainment Expenses Disallows certain entertainment expenses unless directly related to business or bona fide business discussions took place o (without 274, youd just use 162 and argue that taking out clients was O&N for your business) o Test Drinks @ bar with client probably counts Discussion was actively business Talk about business 1st then go out to bar OK Discussion can be preceding or following (need not happen at the actual bar or meal) 274(n) Limitation The 50% rule applies here as well o Food/Beverage o Any item which is a type generally considered to constitute entertainment (Hits both client meals AND travel status) Rationale: o Assume 50% business / 50% pleasure Exceptions: o De minimis fringes ( 132(e)) Ex. Free lunch for Vegas employees Employee can deduct for free lunch on premises under 119 Employer wants full deduction also 274(n) would seem to make it only 50% BUT can qualify as de minimis fringe under 132(e)(2)(B) if: Eating facility is located on or near business, AND Revenue derived from facility equals or exceeds the costs of operating the facility Application If its a regular casino restaurant no problem meeting it If its a stand-alone operation, it might be tough BUT last sentence of 132(e) gets you in

Tax Outline Warren Spring 2007

56 of 104

an employee entitled to exclude the meal under 119 shall be treated as having paid an amount equal to costs. So even if he doesnt pay (and isnt taxed), you can give him meal and that will count as O&N fringe deduction and not a 50% meal (also, cant discriminate)

Activities 274(h) Conventions If convention is held outside N.America, must establish that convention was directly related to the active conduct of trade or business o Factors: Purposes and activities of such meeting / sponsoring organization Residences of the active members of the sponsoring organization o Ex. MD Convention in the Caribbean Might have trouble because its outside N.America and not a great connection between Caribbean and where we typically learn medicine 274(h)(2) Convention on Cruise Ship o Needs written statement from sponsoring organization Facilities 274(a)(1)(B) Facility No deduction is allowed for a facility used in connection with entertainment, recreation, or amusement o BUT food/drink is deductible for event, so most convention spaces will charge you a lot for food/drink and let you use the space for free Dues 274(a)(3) Club Dues No deduction allowed for amounts paid or incurred for membership in any club, even if a lot of business is conducted there o (Law is hard on dues and facilities) What if company paid your dues? o Taxable to them (deduction disallowed as above) o NOT Taxable to you Working Condition Fringe under 132 property or services provided to an employee to the extent that if employee paid himself, it would be deductible under 162 YES these expenses ARE O&N and would be deductible under 162 (even if the deduction would be later disallowed under 274(a)(3))

Tax Outline Warren Spring 2007

57 of 104

What if company pays, but club is 50% biz / 50% pleasure? o Taxable to them ($1000) (deduction disallowed under 274(a)(3)) o Half is taxable to you now ($500) Employees part not deductible under 132 because the deduction is no longer permissible under 162 (not an O&N expense) o Total tax is actually $1500 (even if $1000 consumption)

Meal Moss Lawyers eating together everyday talk about business NOT Deductible meal is personal Other scenarios Cant deduct for your own meal if you go out to lunch by yourself Posners affirmation of Moss o Taxpayers meal would be deductible (subject to 50%) if he ate with client o BUT if hes eating with co-worker, dont need the social lubrication that a meal provides (so the meal isnt necessary) Partner + Client + Associate o Sense after Moss Partners meal taxed (own meal) Client not taxed (but 50%) 162 + 274(n) Associate meal not taxed as fringe (but 50%) o Real World Well let you have all 3 unless its being abused SUM of Entertainment Pretty lax in general o We cant draw a good business/personal line, so we end up using the 50% as a safeguard

RESIDENTIAL PROPERTY
Soliman Anesthesiologist uses part of his home for business use because none of the hospitals he works at give him space to do administrative work SCOTUS disallows deduction for part of home expense Test for determining Principal Place of Business (2) o Relative importance of the activities performed at each location o Hours spent there He was an anesthesiologist office work not exactly his focus Rationale:

Tax Outline Warren Spring 2007

58 of 104

o Kennedy looking for simplicity over accuracy o Concern that people doing minimal amount of work at home will be taking advantage of this 280A Disallowance of certain expenses in connection with business use of home (overrules Soliman) Can deduct now for administrative office if you have no such other place for that o NO deduction if space is used by the taxpayer as a residence o Can allocate portion of expenses for space exclusively used as: Principal place of business for any trade or business, or Used to meet with patients, clients, customers, or Separate structure which is not attached to the dwelling unit, but used in trade or business (admin location counts if you have no other fixed location) ABC Extreme Makeover Home Edition o 280A(g) Limited Rental Use ABC takes over home and doubles its value How to avoid tax? Cant be considered a gift (no detached generosity) Cant be a prize (taxable) So they use this rare provision which allows you to rent part/all of your home for less than 15 days/year and exclude the income Say that ABC rented it and paid in-kind rent (Alt: ABC could just gross them up)

HOBBY

AND

GAMBLING EXPENSES

162 Ordinary & Necessary Taxpayer must show O&N expense of trade or business 183 Activities not engaged in for profit Cant deduct expenses for non-profit seeking activities (hobbies) Presumption of for-profit activity: o If there are 3 profitable years out of 5yr span, then thats considered a forprofit activity Incentive to manipulate income so you get 3 good years, even if the loser years are big o Plunkett You can also prove its for-profit by proving the actual facts Fact-specific inquiry (mud-racing allowed / truck-pulling is not) actual and honest objective of making a profit Concern

Tax Outline Warren Spring 2007

59 of 104

CEO has salary of $1M and hobby expenses of $500,000 o If he can characterize that expense as a business expense, he could deduct and offset his income via personal consumption Goal is for these hobbyists to make their operation look professional o You can enjoy yourself and receive deduction for it up to the amount earned on this activity

165(d) Gambling Losses Allow you to deduct losses to the extent of your gains

EDUCATION
Reg. 1.162-5 Expenses for Education Education expenses are deductible as O&N if they maintain or improve skills needed for employment, but dont make it if: o Education is reqd to meet minimum qualifications for employment o Qualify you for new trade or business (Law students get killed / B-Schoolers dont) Wasenaar Argued that going to get a Masters in Taxation was improving skills he had been using for 10 yrs Court says did it right after the bar, so he was not yet a lawyer who was improving his skills o Test: Need to be in a profession and maintain/improve skills in the same profession If it qualifies you in a new trade or business that doesnt count 222 Qualified Tuition and Related Expenses Can deduct an amount equal to qualified tuition and related expenses Dollar limits: o 222(b)(2)(b) AGI of up to $65k = deduct up to $4k AGI of $65k to $80k = deduct up to $2k AGI over $80k = deduct zero 127 Educational Assistance Programs Amounts paid or expenses incurred by employer for education assistance are excluded Limit: o $5,250 o CANNOT Discriminate

Tax Outline Warren Spring 2007

60 of 104

135 Income from U.S. Bonds used to pay for college If you pay higher education expenses on redemption of bond, your income from the bond is NOT TAXED Rationale: o Incentive for people to go to college Benefit is phased out - 135(b)(2) o If modified adjusted income of taxpayer exceeds $40,000, well exclude from benefit that amount of money which bears the same ratio as the excess over $40k bears to $15k $0k to $40k = full benefit Over $55k = no exclusion 529 State Education Accounts 529 Coverdell Education Savings Accounts Both are exempt from taxation Rationale: o Consistent with thinking about the code as a consumption account Make you pay tax when you consume (but tax-exempt while money is being accrued pre-use for education) 25A Hope and Lifetime Learning Credits Difference: o Hope 1st 2 years of postsecondary education Up to $1500 credit for each year o Lifetime Learning Any course of instruction to acquire or improve job skills of the individual (could include law school) 20% credit for up to $10,000 in expenses (credit) Phase out o For starters Nonrefundable o Then, we will reduce your expenses by whatever amount bears the same ratio to your expenses as your income over $40,000 does to $10,000 Reduction / Expenses = (Excess over $40k) / $10k At $50k, youre totally phased out (bumped to $80k and $100k for couples) No credit allowed if you can take a 162 deduction

JOB-SEEKING

AND

START-UP EXPENSES

Job-Seeking Revenue Ruling 75-120 Expenses in seeking new employment of trade in the same trade or business are deductible if connected with that same trade

Tax Outline Warren Spring 2007

61 of 104

o Seeking a job in a new field is non-deductible Revenue Ruling 63-77 Allowances/reimbursements made to individuals who are interviewing for job are excludable Weird Policy: o Someone paying for your personal expense ( 262), yet we dont tax you o Disconnect between private and public interest: If firm pays, you can get reimbursed without being taxed If you pay for public interest job interview trip, you cant deduct those expenses o Rationale: Maybe suspicion that student not actually looking for job Business Start-up 195(c) Start-up Expenses Two part-inquiry: o Capital vs. O&N o Only certain types of O&N You can only deduct start-up expenses incurred before the start of the business if they would be deductible after as well If Capital Expenditure cant deduct (even if its a start-up) o The threshold for deducting is found in 195(c)(1)(B) The above expenses are only deductible if: if paid or incurred in connection with the operation of an existing active trade or business, they would be allowable as a deduction for the taxable year. Then, if O&N, well allow a deduction for: o Amounts paid or incurred while investigating or creating business o 195(c)(1)(A)(iii) any activity engaged in for profit and for the production of income before the day on which the active trade or business begins in anticipation of the beginning. SUMMARY of the BUSINESS-PERSONAL BORDERLINE Sometimes just categorize as being in one category or the other o Commuting = Personal Sometimes we look at facts and develop and objective test o 183 presumption of 3 yrs of profits out of 5 Sometimes we do motivational analysis under 162 o Plunkett what was his motivation? Sometime we do an arbitrary allocation o 50% rule in 274(n)

Tax Outline Warren Spring 2007

62 of 104

PERSONAL DEDUCTIONS
Not related to the production of income, yet we allow deductions o Business clearly related to production of income o Business/Personal not sure if its related

INTEREST
Personal Interest 163(h) Personal Interest No deduction allowed for personal interest paid during the taxable year BUT with one giant exception: o 163(h)(2)(D) Qualified Residence Interest Let you deduct for home mortgage interest Up to $1M for up to 2 homes Effect: Huge benefit to home ownership o (in addition to imputed income, property tax deduction, and $500k basis adjustment) Benefits wealthy who can buy home Proposed solutions: Allow for rental deduction Reduce the mortgage indebtedness limit Arguments PRO-Deduction of Personal Interest o If we tax interest income, we should allow deduction for interest expense (because interest expense is negative income) o This would treat people the same whether they draw on their own assets for consumption of instead borrow against those assets for consumption ANTI-Deduction of Personal Interest o The effect of allowing the deduction is to encourage borrowing by favoring accelerated consumption o Encourages borrowing and spending earlier Congress is wary of these incentives lately Business Interest Always deductible as an O&N business expense Educational Loans 221 Educational Loans

Tax Outline Warren Spring 2007

63 of 104

Interest on qualified educational loan is deductible up to $2500 Phase-out: o Reduce the $2500 by ratio of: X / 15000 = (Excess salary over 50k) / 50k o (Phase out at $65k) Effect: o Provision aimed at the middle class o Think of the complex interaction in the marketplace we may think were giving taxpayers a credit/deduction here, but maybe this just gives the college an excuse to raise tuition (responsive actors)

Tax Arbitrage Occurs when income from the asset side and the borrowing side are subject to differential tax treatment 265 Expenses and Interest relating to Tax-Exempt Income No deduction is allowed for interest on loans to acquire tax-exempt income o No borrowing to buy tax-exempt bonds Why we have this: o If not, I could borrow money to buy tax-exempt bonds, deduct the interest I paid on my loans and exclude the interest income from the bonds Would give you a negative tax rate Problem: o How to determine what loan interest is allocable to tax-exempt income? The proceeds from which loans were used to buy the tax-exempt bonds? o Tax system uses a combination of tracing and stacking Tracing Borrow money to pay for house deductible Then just use our cash (fungible) to buy tax-exempt bonds o As long as were not borrowing solely to buy the tax-exempt bonds, were OK Stacking If 50% of your funds are in tax-exempt bonds, maybe we should only allow a 50% deduction Criticism: o Creates windfall to high-bracket people who buy the tax-exempt bonds When there arent enough people with their own money to buy taxexempt bonds, the govt must RAISE the interest rate to attract lower-bracket people Ex. o Mkt interest rate on normal bond = 10%, but subject to 35% tax rate, thats 6.5% return

Tax Outline Warren Spring 2007

64 of 104

o Since you dont pay any tax on the interest on a govt bond, they can offer only 6.5% interest and allow buyer to achieve the same result o If there arent enough buyers at 6.5%, however, the govt must raise the interest rate to get people to buy it 7%, 7.5%, etc. Result: This is a pure windfall to high-bracket people since a normal bond is paying 6.5 and the tax-exempt bond is now paying more If 265 were repealed, high-bracket people would have the opportunity to borrow to purchase tax-exempt bonds, so cities could pay lower interest rates and wed no longer have this windfall to the rich The system perpetuates the windfall to avoid the appearance of unfairness that would result if high-income people owned all the govt bonds Interest disallowance provisions permit wealthy taxpayers to obtain relatively greater benefits from tax-favored assets because they can acquire tax-favored assets by liquidating their existing assets

3 kinds of tax arbitrage we allow inconsistency o Home ownership Exclusion of imputed rental income + deduction for interest paid on home mortgage interest o Depreciable Property Depreciable assets are subject to less then full taxation because you can depreciate the machine, but you get a deduction for borrowing to buy a machine (O&N) Accelerated Depreciation + Interest Deduction = Negative Rate o Current Expensing of R&D We allow current expensing for R&D if you borrow to engage in R&D and deduct the interest on that borrowing, thats negative effective tax rate too

Limits 163(d) Limits on Investment Interest Limits deduction for investment interest to the amount of investment gains for the year o Prevents you from borrowing to invest and deducting the interest costs, but deferring the tax on your gains until you sell (No deduction unless you realize gains in that year)

Tax Outline Warren Spring 2007

65 of 104

263A(f)(1) Capitalization and inclusion in inventory costs of certain expenses Interest paid or incurred during production period must be capitalized Interest allocable to capital assets must be capitalized o You must capitalize interest if youre producing something and it hasnt produced income yet o Cant deduct interest to get deferred benefit of construction Arbitrage Chart
Taxexempt bonds Income (less than full taxation) Deduction of Interest Interest from bonds is exempt No deduction Home Depreciable Assets Accelerated depreciation Investment Income Deferred Gain Construction Period Interest Deferred income from construction Interest capitalized if hasnt produced income yet Tough

Imputed rental income excluded Full deduction

Full deduction of interest (O&N) Easy

Limited Deduction (only to gains) Tough

Treatment of Tax Arbitrage

Tough

OK (you can scam)

Sham Transactions Knetsch Taxpayer borrowed $4M at 3.5% interest in order to purchase annuity bonds from Sam Houston Life Insurance that would pay 2.5%. He then borrowed against the annuity bonds and prepaid interest each year o (were immediately suspicious that hes earning at 2.5, but paying 3.5) o Total LOSS before taxes, but if the interest on the loans is fully deductible, then he has after-tax GAIN Hes getting taxed at special cap gains rate for the 2.5% inflow, whereas the outflows hes paying out are taxed higher (whats earned is taxed at low rate, whats loaned out is taxed at high rate) Court disallows interest deductions stemming from a sham transaction with no commercial economic substance and no effect on the taxpayers beneficial interest except to reduce his tax. o Ignoring tax, there was no way this annuity was ever going to be profitable for Knetsch Dissent: (Douglas) o Transaction was perfectly legal o Tax avoidance is an acceptable motive for economic transactions

Tax Outline Warren Spring 2007

66 of 104

IRS Solution: o Many transactions occur for tax reasons! o IRS just looking for at least some reasonable RoR in the pre-tax column (some valid reason for moving forward)

Goldstein Irish Sweepstakes winner paying 4% to borrow money to purchase Treasury notes paying 1.5% o Prepays interest and tries to deduct that payment to offset the $140,000 gain from the sweepstakes Court holds that even if transaction is not a sham, the interest deduction can be disallowed if there is no purposive reason, other than the securing of a deduction. o Avoid transactions with no economic utility other than tax benefit o Want PRE-TAX profit! SUMMARY on INTEREST Three types: o Business (deductible) o Personal (generally non-deductible) o Investment (deductible within limits) Up to gains Real transaction No tax arbitrage Knetsch and Goldstein show that IRS can disregard a transaction that is legal in form provided its substance is shady

TAXES
164 Taxes You can deduct following taxes from your income: o State, local, and foreign real property tax o State and local personal property tax o State, local, and foreign taxes on income, war profits, and excess profits Rationale: o Defining Income Gives a more accurate view of what youve earned dont have that money for consumption Avoid double taxation Counter: o Youre consuming state/local services

Tax Outline Warren Spring 2007

67 of 104

Regressivity Issues The deduction helps high-income a lot more than low-bracket people o How? Assume flat state tax rate of 10% BUT that 10% STATE rate reduces the FEDERAL income tax burden differently for different tax-payers If a low-income person (15% tax bracket) gets to deduct 10% of income, then he pays less federal taxes o Effective STATE rate for a 15% FEDERAL payer: 8.5% If a high-income person (40% tax bracket) gets to deduct 10% of income, he also pays less federal taxes, but the benefit to him is greater because being able to deduct money means more to him (if he had to include it, hed get taxed more) o Effective STATE rate for a 40% FEDERAL payer: 6% Lower effective STATE rate (the state income tax is not really flat its regressive) Incentives Gives Governor incentive to export taxes to federal government by RAISING the state tax (can reason to constituents that theyll be paying less in federal as a result) LOWERING state taxes results in population incurring MORE in federal taxes so citizens do not receive the full benefit of a state tax-cut Benefit to Homeowners Another tax benefit to homeowners over renters since it effectively reduces your property tax o BUT landlord cant pass this benefit onto tenant because: Because LL can deduct $4k of the $10k tax (assume t=40%), the tax only costs him $6k But to recover that $6k after-tax, he needs to make $10k in additional pre-tax rent (so hes implicitly taxing the tenant) o SO the only way to capture this benefit is by owning your home and not renting (homeowners shift burden to renters) NY tries to get around this by saying that renter actually paid the tax and should be allowed to deduct o Rev Ruling 79-180 disallows this Tenants cannot deduct payments of property taxes passed along by landlords as provided by state statute 3 ways to provide federal subsidy to states Grants in aid

Tax Outline Warren Spring 2007

68 of 104

o States dont love this political process every year State-Local Tax Deductibility Exemption of interest on tax exempt bonds o Less efficient dont get full benefit for each dollar of federal costs

CASUALTY LOSSES
165(c)(3) Casualty Losses You can deduct losses resulting from fire, storm, shipwreck or other casualty o Loss determined based on adjusted basis (lesser of FMV and Basis) Test: o Must be sudden, unexpected, violent, unusual not something you can foresee/alleviate Termites No Diamond ring with violent blow Yes Administrability Limit o Casualty provision is only invoked for very big losses 165(h)(1) Casualty loss must exceed $100 for event 165(h)(2) Losses only deductible to the extent of casualty gains + anything over 10% of AGI Rationale for deduction: o Measure of Market Loss (true income) Distinction between market loss and decline in value due to nontax imputed income from ownership If you drive your car for 5 years and down in value NO If you crash your car deduction o Subsidy But if we wanted to do a subsidy, isnt that best achieved via a credit? A deduction gives a bigger subsidy to the rich because they benefit more (higher-bracket) Counter: o Disincentive to buy insurance?

MEDICAL EXPENSES
213 Medical, Dental, Etc. Expenses Allow deduction for expenses paid during year for taxpayer, spouse, or dependent that are NOT compensated by insurance o Diagnosis / cure / mitigation / treatment / prevention o 213(b) drugs only covered if theyre prescription or its insulin o Borderline cases:

Tax Outline Warren Spring 2007

69 of 104

Swimming Pools can deduct to the extent the expenses incurred exceed the subsequent increase in the value of the house Food / Toothpaste / Maternity clothes NO (may technically prevent disease but) Plastic Surgery explicitly excluded from deductability (213(d) (9)) Movie stars may try to get in as O&N expense Administrability Limit: o Can only deduct for expenses that exceed 7.5% of AGI o Rationale: Limit to losses you cant control so we get a truer estimate of your income

Policy Discussion Current: o Employer provides insurance excludable for employee under 106 o Employee buys own insurance only excludable if expense over 7.5% Discriminates against people who buy own insurance Incentive for employee to overbuy insurance since employer is paying for it and its excludable Bush Proposal: o $15k Deduction for every taxpayer if employee or employer buys insurance in any amount Eliminates the discrimination depending on who buys BUT now it incentivizes buying the cheapest possible policy o Business community likes this because now employers wont be under the same pressure as before to provide healthcare Now: employee wants the goldplated policy because its deductible o BUT if American businesses get rid of healthcare (or provide lesser coverage), theyll have to pay more in wages to the employee Employee costs some amount $X to the employer that will be allocated between wages and health insurance Pay less healthcare pay more in wages Business community thinks this: Pay less healthcare (pay same wage) pay less total o Goal: Will encourage people to buy more reasonable healthcare plans and the $15k deduction might provide more tax savings How to redesign if youre a Dem: o Make it a refundable tax credit (would help low income people)

CHARITABLE CONTRIBUTIONS
Rationale

Tax Outline Warren Spring 2007

70 of 104

Subsidy to charitable organizations (encourages donation to these groups) Its not your consumption (Haig-Simons)

170 Charitable Contributions and Gifts Charitable contributions are deductible Limit: o 170(b) 50% of individual donors income (or 10% for corporations income) o 170(c) Deductible only if donations made to: State if used for public purpose Charity if purpose is OK under 501(c)(3) War Veterans organization Fraternal organization if purpose is OK under 501(c)(3) Cemetery company under certain circumstances o Contribution must be made to an institution, not an individual Can try to work around this by directing the donation in specific ways, but its legally and ethically questionable Inequity: o Cheaper for high-income people to give (deductions worth more to them) [combined below with Double Benefit to rich mans charities] ALSO, we will see below that deductions under 170 are below the line so youll only be able to claim them if you itemize Hernandez Quid pro Quo you can only deduct what you give in excess of whats received Scientology Case o IRS says he was essentially purchasing something and therefore he cannot deduct the full amount of the donation o Taxpayer claims he received something, but says he should still get a donation because other religions make you pay for something and youre able to deduct Church pews, Jewish high holiday tickets Court says NOT DEDUCTIBLE because payments were made primarily for personal accommodation o Distinguishes this from church pews by nothing that these charges/benefits were directly personal as opposed to indirect contributions made in other religious contexts (Court generally very generous to religious organizations only one that ran into trouble was Bob Jones due to racially discriminatory practices) Property Donations POSSIBLY LIMITED 170(e) Contributions of Ordinary Income or Cap Gain Property Goal: o You want to be able to deduct the whole value without having to pay tax on the gain

Tax Outline Warren Spring 2007

71 of 104

For instance, if we treat you as if youve sold it and then given the cash, that gift is not privileged (buy at $30, sell @ $100 = $70 gain; if youre first taxed on that $70 gain and then deduct the $100 donation, your net deduction is only $30 o Charities want donors to be able to deduct all $100 Double Benefit!!! Increase in value isnt taxed on either party Statutes Limits: o 170(e)(1)(A) property that would have provided ordinary income no double benefit o 170(e)(1)(B) Favored Gifts The amount of charitable contribution will be reduced by the amount of long-term capital gain if the property had been sold by the taxpayer at its FMV BUT this only applies to: tangible personal property, if the use by the donee is unrelated to the purpose or function constituting the basis of its exemption under 501. Applied: Securities exempt (not tangible) Real Estate exempt (not personal) Old Medical Tools not sure (they are personal and tangible, but if the use is related to what youre donating to, youre fine) o All of these get the Double Benefit treatment Effect: o Rich people more likely to donate appreciated property, so rich charities (Metropolitan Opera, Harvard) are the ones who benefit from the double deduction o Other charities are usually given cash, so no help (Red Cross, church) o Argument: Rich charities claim that the alternative to allowing this Double Benefit where the increase in value isnt taxed to either party is that the donor will just die with it instead of being forced to realize the appreciation Step-up basis at death under 1014 (System will never see the money anyway)

Bargain Sale to Charity 1011(b) If bargain sale to charity, well only let you use up whatever portion of YOUR basis as matches the ratio of the AR to the FMV o If you sell it to a charity for of FMV, you can use of your basis (taxed on rest) Vertical rather than Horizontal division of property

Tax Outline Warren Spring 2007

72 of 104

Services No deduction of services Exemption from Tax 501(c)(3) Exemption from Tax on Corporations, Certain Trusts, etc. Exempt from taxation if you meet the statutory requirements for type/purpose No part of the earnings may inure to the benefit of any private individual No lobbying or participation in political campaigns Charitable Contribution vs. Gift 170 is better than 102, so pressure to structure gift as charitable contribution

Tax Outline Warren Spring 2007

73 of 104

COMPUTATION
TAX RATES

OF THE

TAX

1 Tax Imposed Graduated Rates Rates depend on filing status (single, married, etc.) Difference between marginal and average rates

TAXABLE INCOME
How to do this: 1. 61 Gross Income Start by taking everything into account 2. 62 Adjusted Gross Income There are certain deductions known as adjustments under 62 that you get to take regardless of whether you itemize in Step 3 (Above the Line) o You take the 62 adjustments no matter what! o Types: (non-exhaustive) Trade or business expenses (though not if the trade or business consists of performance of services by employee) Reimbursed expenses of employees Expenses of performing arts, officials, elementary/secondary school teachers, members of reserve components of armed services Losses from sale or exchange of property Deductions of life tenants and income beneficiaries of peropert Pension plan of self-employed individual Retirement Savings Alimony Required unemployment payments Moving expenses deducted under 217 Educational Loan interest payments under 221 Higher Education expenses under 222 3. 63 Taxable Income {This is where you get a choice) o 63(a) Itemization Subtract Personal Exemptions - 151 Subtract Itemized Deductions

Tax Outline Warren Spring 2007

74 of 104

63(d) all deductions other than the 62 deductions and the 151 personal exemption o Charitiable contributions are below the line o 63(b) Standard Deduction Subtract Personal Exemptions - 151 Subtract Standard Deduction 4. Apply the applicable tax rate according to Taxable Income and Filing Status (see page ix) Standard Deduction and Personal Exemption 63(c) Standard Deduction Only get to take if you dont itemize Simply deduct that from your AGI o 2007: Married filing jointly = $10,300 Unmarried = $5,150 151 Personal Exemption Get to deduct flat amount for taxpayer, spouse, and each dependent o 2007: $3300 Limits on Deductions 67 2% Floor on Miscellaneous Itemized Deductions Miscellaneous itemized deductions shall only be allowed to the extent that these types add up to over 2% of AGI o Miscellaneous = ALL TYPES OTHER THAN: Interest ( 163) Taxes ( 164) Casualty Losses ( 165) Charitable Contribution ( 170) Medical Expenses ( 213) (and others) Rationale: o Simplification (encourages less itemizers) o Revenue Incentive: o Stockpile deductions in a single year rather than spread out 68 Overall Limitation on Itemized Deductions Itemized deductions decreased by 3% of the excess of AGI over some applicable amount ($150,500 in 2007)

Tax Outline Warren Spring 2007

75 of 104

o BUT Max phaseout is 80% of your itemized deductions 151(d)(3) Phaseout of Personal Deduction If youre over some threshold amount, you start to lose some of your personal exemption o 2% down for every $2500 over the applicable amount Effect of these deductions o Congress doesnt really have the personal courage to raise the tax rate, so instead they do backhanded things like this and take away some of your deductions effectively raise your tax rate, but in a less politically dangerous way More money leads to: Greater tax liability to begin with (higher income = more tax) Reduction in personal exemption (152(d)(3)) Reduction in itemized deduction (68) o (Congress is in the process of phasing out the reductions in personal exemption and itemized deductions) For this year, theyre only reducing by 2/3 of otherwise (if you were previously reduced by $150, youll know only be reduced by $100)

ALTERNATIVE MINIMUM TAX


Rationale: o Make sure that people pay taxes o Reflects ambivalence in Congress between those who think we should have deductions like we do and those who think the current system is just fine So we give them to you, but take away if you get too many Controversy: o Levels have not been adjusted for inflation, so tons of people have been subject to it lately Congress hesitant to change because they need the revenue

55 Alternative Minimum Tax imposed Rates are lower (26 or 28%) BUT Base is considerably higher (few deductions) o Rates: 26% of 1st $175,000 28% of over $175,000 o Calculations (under 56(b)(1)(A)) Miscellaneous itemized deductions Interest, taxes, casualty/theft losses, 501(c)(3) contributions Taxes from 164 Hits high-tax states hard (coastal states Dem states)

Tax Outline Warren Spring 2007

76 of 104

Medical expense threshold raised to 10% Depreciation deductions go from Double Declining to 150%

PERSONAL TAX CREDITS


Credit is considerably more valuable than a deduction 32 Earned Income Tax Credit Rationale: o Want to use the tax system to help lower income people who are working If youre below a certain income level, well give you some credit on a % of your income o (This is not welfare because you have to work in order to get the credit) How to calculate: o If AGI is anything up to the Earned Income amount, you get a % back based on how much you earn up to that amount (amount adjusted for inflation) Percentages depend on # of qualifying children 0 children 7.65% 1 child 34% 2 children 40% o If AGI is anything between the Earned Income amount and the Phaseout Amount (plateau), you simply receive the max earned income amount Plateau = Applicable Percentage x Earned Income Amount o Once AGI goes beyond the Phaseout Amount, we begin subtracting from the amount of credit we calculated at the Plateau based on a percentage of AGI Percentages depend on # of qualifying children 0 children 7.65% 1 child 15.98% 2 children 21.06% o So if you have one kid, once you get past the 2nd amount, you start to deduct 15.98% of your income from the Earned Income amount o At some point, the phaseout will eclipse the Earned Income amount and your EITC will be reduced to $0 The credit is refundable o Not just a reduction in taxes, but an actual outlay Problem: o People with such little income dont generally file tax returns anyway, so they may not know about the EITC o Solution: Congress allows employer to have negative withholding, though theres some potential for fraud with this 24 Child Tax Credit

Tax Outline Warren Spring 2007

77 of 104

For every kid, you get $1,000 nonrefundable credit o In addition to the personal exemption o A family of 4 with 2 kids can earn up to $41,866 without paying tax Personal Exemption + Standard Deduction + Child Tax Credit Phase out: o 24(b)(2) Phase out credit $50 for each $1k over the threshold amount of $110k (out at $130k) Rationale for the credit: o Want people to pay what they can afford to pay (if you have children, might not be able to afford it) Policy o The credit is actually worth more now that Bush has lowered tax rates Under old rate (15%), you were off the tax rolls if you make under $6,666 Under new rate (10%), youre off the tax rolls once you make under $10,000

Tax Outline Warren Spring 2007

78 of 104

CHOICE

OF THE TAXPAYER Given the graduated rate structure, there is an incentive to shift income to a lowerbracket person from a higher-bracket earner o Now it matters who earns the money

THE TAXABLE UNIT


Marriage Druker Court upholds marriage penalty against an EqPC challenge o Determines it is a question for the political branch to decide
A Individuals Income Pre-1948 Policy 1 50 M B 1 50 M 2 0 -C 1 50 M 2 50 M D (Comm Prop) 1 2 100 0 M (50) M (50) E (Non-Comm Prop) 1 2 100 0 H --

Tax Individuals Married couples said to split income Lower the Difference between single & married Credit the 2earner couple Revert to 1970 rules - (no 2earner credit)

1948

M (50)

M (50)

M (50)

M (50)

1970

L+

M (50)

M (50)

M (50)

M (50)

1982 1987 to Now

L+ L+

L L

L L

MM

MM

M (50) M (50)

M (50) M (50)

M (50) M (50)

M (50) M (50)

Problems over the years Pre-1948 E is mad because hes in an identical position as D, but community property rules help D o Argue that theyre paying more than other 2 married couples making the same amount 1948 A is mad because a married couple with the same income (B) is doing better o Single mad (he works with B1, hes getting taxed more 1970

Tax Outline Warren Spring 2007

79 of 104

Marriage Penalty o A + A = C, yet C pays more than 2 As

1982 Weve given a small credit to the 2-earner couple, but now weve pissed off D & E o Argue that C, D, and E all making same as household, yet C doing better o Why might we like this? Incentive to work D & E getting benefit of non-taxation of imputed income 1987 Revert back to 1970 and the Marriage Penalty case PROBLEM: With graduated tax rates, we cant have all married couples pay the same tax AND solve the marriage penalty problem If rates are graduated and there are no incentive effects, then we cant tax all couples the same If rates are graduated and we tax all couples the same, then there are incentive effects (like now!) o A gets hurt from getting married A + A < C o C gets hurt vs. D & E because no imputed income help Marriage Penalty Two individuals earn $154,800 each o Tax = $37,675.50 each $75,351 (single) Get married same income = $309,600 o Tax = $82,149.50 Difference is $6,798 (less to squander on living) 2.2% Marriage Bonus Theres actually a marriage bonus for the highest-earning individuals o Single man making $336,550 Tax = $97, 653 o Gets married to $0 spouse Tax on couple = $91,043 After-Tax Difference of $6610 1.96% marriage bonus (The difference is that this is a one-earner couple BONUS) Possible Solution Tax individuals separately ignore community property laws imposed by states o If you dont work and your husband makes big bucks, tax you him big and you not at all Problem:

Tax Outline Warren Spring 2007

80 of 104

o Would have to monitor arrangements where the high-earning spouse in D & E would have incentive to try to disguise income as being earned by the lowincome spouse Same-Sex Marriage Congress has passed DOMA, so same-sex couples can neither suffer the penalty nor receive the bonus o Normally, however, we defer to state law on the marriage question Divorce Review of basis for gifts and exchanges Gift (1-asset exchange) o Carryover basis from whatever the donors basis was ( 1015) o (only get step-up basis at death) ( 1014) Exchange (2-asset exchange) o To determine your AR, you take value of what you received minus the basis of what you gave up: AR new B old o The exchange is a realization event taxed o Post-exchange basis = cost ( 1012) Farid es-Sultaneh Wife giving up right to his property in the event of divorce in exchange for his stock What is the basis in the stock? o If we characterize this as a gift, then she gets carryover basis and thats very low Govt position o But if we characterize this as a valid exchange, then she gets the high basis at the time of the transaction Her position Court determines this was not a gift, but rather a valid exchange of her dower rights in exchange for the stock o Under courts logic: Maybe she should have been taxed on $100 when the deal was made FMV of the Stock vs. the rights she gave up BUT we dont tax human capital (value of being released from POW camp) Maybe he should have been taxed on something at the time of exchange at least (appreciation in stock) 1041 Transfers of property between spouses incident to divorce

Tax Outline Warren Spring 2007

81 of 104

No gain or loss shall be recognized on transfer of property to spouse or former spouse if incident to divorce (made within one year of end of marriage) o No realization event No Tax 1041(b) Dealing with Basis o Treat this as a gift Transferee takes Transferors Basis (assuming husband doing the transferring, this hurts the wife because now she has to pay taxes on the appreciation) How to maneuever post-Farid and 1041: o 1041 only applies to a spouse or former spouse o You want to make the agreement after the ceremony because before the ceremony, 1041 does not yet apply if you do it after, then 1041 applies and theres carryover basis

Alimony and Child Support 71 Alimony 215 Alimony Payments Taxpayer allowed deduction for alimony payments - 215 Alimony must be included by recipient Taxed o Parties can contract around this within 71(b)(1)(B), but the default setting if the divorce agreement is silent on the issue is that the spouse that pays can do the deducting They will usually pick the lower-bracket spouse Private Ordering 71(c) Child Support 215 Child Support Payments No deduction for taxpayer under 215 Child Support not included in income under 71(c) o Incentive for payer to frame payments as alimony instead of child support Creates substantial family law problems

ASSIGNMENT

OF

INCOME

1(g) Kiddie Tax When child is under 14, childrens income from capital is taxed at higher of childs rate or parents rate o Ordinary (paper route) income taxed at childs rate Rationale: o Prevent parties from shifting all their capital income to their dependents and paying lower tax rates on it

Tax Outline Warren Spring 2007

82 of 104

Gift of Income from Labor Lucas v. Earl Husband is employee, but tries to have contract such that he is the sole performer, but half of income goes to wife o Split up and put each in lower bracket (this was pre-1948 and community property assumption) Court rejects all salaries must be taxed on those who earn them o Cannot stand for arrangement where the fruits are attributed to a different tree from that on which they grew For Love of Money 2 Erin wins and decides to split prize with Chad Should it all be taxed to her? o She was the one doing the earning; she provided the services o She gave it to him as a gift (not tax on him Gift of Income from Property Parking Lot Hypo Father pledges to give all the revenue from a parking lot he owns to his daughter We still tax the father o He has control over parking lot o He is still the principal o He still owns the property He is the tree! the payments to her were gifts, but he must pay tax on the income If he gives her the property, then she would have to pay tax on the income o (If she were only 5 years old, wed be wary Kiddie Tax) Horst Bond Dad controls/retains principal, but gives coupons to sons to get the interest Interest is taxable to Dad o He is still in control of the tree he controls income, so hes taxed Think about Horst vs. Irwin o Horst A gives income to B for a term, remainder to A tax donor o Irwin A gives income to B for a term, remainder to C tax donee Irwin codified at 102(b) which says donee is taxed BUT Reg. 1-102-1(e) says if we tax donor, then you can ignore 102(b)s mandate to tax donee someone already taxed Under this, we wouldnt tax the donee in Horst o Why the distinction between Horst and Irwin? Irwin remainder went to a 3rd party Horst remainder back to A (more potential for abuse)

Tax Outline Warren Spring 2007

83 of 104

Blair Trust beneficiary assigns his interest in the trusts income to his children Interest is taxable to Children o Dad has only limited control of the trust; never getting remainder of trust (income to taxpayer for life, remainder to grandchildren) Hell never see that tree again (all equitable rights given to kids) o The court classifies the interest in the trust income as property so its tax treatment is like that of a gift of property (gave away the parking lot) Whats the difference between Horst and Blair? Blair gave away everything (gave up the tree) o Horst carved out rights (held onto the tree) Blair was a more substantial transfer o Big slice vs. Small slice Legal rights in Blair rose to the level equitable rights (court likes this one) o Gift in Blair was effectively a gift of the entire property the donor owned equitable o Gift in Horst was only the interest (fruits) and not the bonds (tree) legal Note: Equitable rights entitled to specific performance (so if Blair reneges on gift, children might be able to enforce) Comparing Assignment of Income from Property vs. Income from Services Incentive to classify what youre assigning as income from property turn income from service into income-producing property o Ex. Write a book If you sold the book yourself, that would be income from services and non-assignable But if you own copyright and sell that, you can assign income that way

OTHER ARRANGEMENTS
7872 Interest-free Loans For a gift loan or demand loan with below-market interest rates, the foregone interest is deemed to be: o (A) transferred from lender to borrower (and not characterized in any way), then o (B) re-transferred (as interest) from the borrower to the lender Loan from Parents o (A) Transfer of foregone interest from lender to borrower (Parent to Child) Parents TAXED (nondeductible gift) Child Not Taxed (gift) o (B) Imputed transfer back (Child to Parent) Parents TAXED ( 61)

Tax Outline Warren Spring 2007

84 of 104

Child TAXED (unless secured by a home) Double Taxation here o If the the child invests the principal, hell earn interest and be taxed on it also o 7872(a) creates fiction that the loan was made at the market interest rate Loan from Employer o Common move for employer to entice employee (Many employees use the loan to buy a house) o (A) Transfer of foregone interest from lender to borrower (Employer to Employer) Employee TAXED (salary) Employer Deduction ( 162 deduction for salary) o (B) Imputed transfer back (employee to employer) Employee Deduction (Home mortgage interest) ends up at 0 Employer TAXED ( 61) o If the transaction is done right, neither the employer nor the employee has tax consequences Great Deal employee getting more salary (and using it to pay mortgage interest) SUM o 7872 treats below-market loans like transfers of interest income (fruit) where transferor retains principal (tree) Transferor is taxed on the interest income (Horst)

691 Recipients of Income in respect of decedents Any gross income of decedent not includable in decedents income in year of death is included in income of the recipient o Compare Labor and Capital Labor (wages) If employee owes you at death, taxable to estate or donee Capital Transfer of property at death results in stepped-up basis o Other places where we treat labor and capital differently Charitable Contributions Labor not entitled to deduction Capital possible to get double benefit for favored gifts o (neither side realizes appreciation) Kiddie Tax Labor taxed at childs lower rate Capital taxed at parents higher rate

Tax Outline Warren Spring 2007

85 of 104

WHEN
GAINS

IS AN

ITEM TAKEN
ON THE

INTO

ACCOUNT?
OF

AND

LOSSES

DISPOSITION

PROPERTY

CONSTRUCTIVE REALIZATION
Short Sales Against the Box Technique: o I want to cash out (monetize) securities without paying taxes on the gain yet o I can borrow identical securities against the ones I have now o I sell the borrowed ones, but dont yet pay tax on the sale because I wont know the basis in the stock sold until I repay the loan by giving the lender my original securities Result: o Whether the asset increases, decreases, or doesnt change in value by the time the loan is repayable, Ill have shifted my taxable gain from the present to that time Estee Lauder Facts: o Owned stock with low basis and FMV of $50M o When she was 87, she wanted to walk away with $50M and pay no tax o She borrowed stock of her company with a FMV of $50M from her family and sold it She sold short (sold something she borrowed) She was also long (continued to own her own stock) o Where we are now: We know basis in her own stock (low), but not AR (because she hasnt sold that stock yet) We know the AR in her familys stock ($50M that she just sold it for) but dont know the basis (she owes them specific # of shares down the road) o Estee now has $50M to squander in life without any tax consequences Shes imported 1014 step-up into real life, because at her death, two things would happen: Her family would get her own stock but with step-up in basis to todays FMV Her executor would use her stock to pay back the loan owed to the family anyway o Estee will have no tax liability now or upon death Forward Sale How it works: o A & B agree that A will sell property to B in the future

Tax Outline Warren Spring 2007

86 of 104

Why it matters: o You no longer have the risk that the property will change in value (youve effectively sold it today but deferring the tax to future) Incentive to do this if you have low basis

1259 Constructive Sales Treatment for Appreciated Financial Positions (Radical departure from the normal realization requirement) If there is a constructive sale of an appreciated financial position, the taxpayer shall recognize gain as if such position were sold, assigned, or otherwise terminated at its FMV on the date of the constructive ale o Appreciated Financial Position Any position if there would be gain were such position sold o Constructive Sale Types - 1259(c) Short sale Offsetting notional principal contract Futures or forward contract substantially the same effect as whats listed here o Tax Deferral o No exposure to risk o Monetized Investment Rationale: o Prevent short sales against the box and similar maneuvers Issue o How much downside risk or upside gain to limit is too much? When will your sale be classified as a constructive sale?

DEFERRED PAYMENTS
Open Transactions Burnett v. Logan Taxpayer sells stock for cash + right to receive 60 cents for each ton of ore removed from a certain mine Where a sales price is contingent on some future event, the seller is entitled to recover all of his basis up front o This is very favorable to the taxpayer (lose basis first and defer any gain) Great incentive to structure transactions this way o Govt doesnt like this because of the deferral Rationale o Liquidity if its really contingent, they might not have money to pay the tax right now Alternative the govt might propose: o Estimate the gain and make that the AR (govt does estimates all the time!)

Tax Outline Warren Spring 2007

87 of 104

Installment Sales Disposition where at least one payment will be made after the close of the taxable yr 453 Installment Method Income for any taxable year is that proportion of payments received that bear the same proportion as gross profit bears to contract price o So if youre making a 50% profit on the deal as a whole, then 50% of what you make in that year is taxed as gain Apportion the gain according to the AR (You can elect out of this - 453(d)) Exception for related parties: o 453(e) If you sell to a related party via the installment method and the related person makes a subsequent disposition less than 2 years after the 1st sale, then you treat the AR in the 2nd sale as AR by 1st owner Hypo: Dad has $5k basis but sells to daughter for $100k to be paid in 10 yrs she turns and sells it for $100k next week o Well nail dad on $95k and daughter on $0k (because her basis was stepped up when she bought it Installment Method and Interest Sells property today ($5k basis) in exchange for $10k to be paid in 3yrs (interest free) o Gain will be $5k in 3 yrs Problem: o Seller is basically lending money to buyer by selling w/o interest BUT the system taxes interest differently ordinary income vs. capital gains which are lower Solution: o Take the PV of a $10k payment in 3 yrs $7514 (i = 10%) o Convert the AR to $7514 (that gives us a $2514 capital gain) Leaves $2476 to recover in interest payments o Then collect interest payments compounding: 1st yr owes $751 ($7514 + $751 = $8265) 2nd yr owes $826 ($8265 + $826 = $9091) 3rd yr owes $909 ($9091 + $909 = $10k) Total interest payments = $2486 $5k is total gain o [If we let you report the way you want: $5k capital gain, $0 interest NO] Instead: $2514 cap gain, $2486 interest 483(a) o Treat as interest that portion of the total unstated interest as determined in a manner consistent with 1272

Tax Outline Warren Spring 2007

88 of 104

Figure out PV of contract anything else is INTEREST and well make you allocate that on a compounding basis over the period Well let you defer the gain, but not the interest Must claim interest income in the first 2 years and then interest + capital gains income in the 3rd year

NONRECOGNITION
1031(a) Like-Kind Exchanges Wont tax you for exchange of property that is alike o Does NOT apply to stocks Microsoft exchange for Google is not cool o Gives incentive to find 3rd party to help do deal and cut corner BUT 1031(a)(3) requires that the like-kind exchange be carried out in less than 180 days What if theres a boot? o [Youre giving away property with FMV 110 in exchange for property with FMV 130, so the other guy needs you to throw in a $20 painting to boot] o 1031(b) If an exchange would be within 1031(a) if it were not for the fact that the property received in exchange consists not only of like-kind property, but also of other property (i.e. a painting) or money, then the gain shall be recognized, but only in an amount not in excess of the sum of money or the FMV of the property Most gain you can recognize is the money received or FMV of property received Rationale: o Not changing your investment Basis of New Property owned: o 1031(d) Basis shall be that of property exchanged (your property) decreased by amount of money received by taxpayer and increased by amount of gain & decreased by amount of loss You exchange FMV 110 property (basis = $50) for FMV 110 Property (basis = $70), then your basis is still $50 no matter what the basis (in the hands of the other party) was of the property youre getting

Basis Now = Old Basis + Gain Loss Money Received


o Boot Basis Cash Boot If youre the boot recipient, its fairly easy because you properly claim the cash received as a Gain under 1031(b) and then subtract it right back out as Money Received If youre the boot giver, you take your old basis and then add in whatever cash you just gave

Tax Outline Warren Spring 2007

89 of 104

o Higher basis, so lower tax liability later; makes sense because you paid cash now Non-Cash Boot (Assume you throw in a $20 FMV painting that has a $25 Basis in order to equalize the transaction) Since its not a like-kind exchange wrt the painting: o The boot giver would recognize a LOSS of 5 on the painting under 1001(c) o The boot recipient would recognize a GAIN of 20 on the painting The basis for each party would be reflected as follows: o Boot Giver Start with Old Basis in the main property, add in basis of the boot, then subtract out the loss recognized on the boot o Boot Recipient Start with Old Basis in the main property, then add the gain recognized on the boot (but this gives total basis in property + boot and you can split off the boot by giving as basis its FMV)

1033 Involuntary Conversion Wont tax you for compulsory or involuntary conversion of property into property similar or related in service or use o 2 yrs o 1033(h) Presidentially declared disaster 4 yrs Rationale: o No choice! New Basis: o 1033(b) Same as property replaced 1044 Rollover of stock gain to small business investment Wont tax you on gain from sale of Microsoft stock to the extent you are investing the AR in a small business Rationale: o Saying you havent really gained o Some preferential policy treatment for small businesses 121 - Exclusion of gain from sale of principal residence Provides exclusion of up to $250k ($500k for married couple) for gain on sale of your principal residence o Can be taken every 2 years Rationale: o Without this, homeowners with hugely appreciated homes would hold onto them near death because they wouldnt want to suffer tax consequences before 1014 basis step-up kicked in

Tax Outline Warren Spring 2007

90 of 104

This is the 4th major home-ownership benefit: o Non-taxation of imputed rental income o Home mortgage interest deductibility o Property tax deduction o $500k exclusion for principal residence appreciation

LEVERAGE

AND

DEFERRAL

Recourse vs. Non-Recourse Borrowing Recourse Borrowing o Borrower is personally liable for repayment of the debt o Lender can go beyond the property (he has recourse) Upside gain and Downside loss on the borrower still Non-Recourse Borrowing o Borrower NOT personally liable o Lender can only look to assets that secured the debt payment No downside loss for borrower o [This type looks more like a sale no downside loss for taxpayer] Estee Lauder gave up both upside gain and downside loss, so we held her to a constructive sale Crane Facts: o Crane inherits property with FMV of $260k which is subject to debt of $260k (real-world worth is $0) o He takes $28k in depreciation deductions o Sells property for $2500 and tries to take just a $2500 gain Arguments: o Crane wants to use equity basis compute basis only on taxpayers own contributions BUT hes taken depreciation deduction to reduce his taxes in the past and has now fooled the system into believing he had a loss o The IRS wants to say: Basis was $260k (including what he borrowed), which was then reduced to $232k via the depreciation deductions AR was $262,500 ($260k from amt borrowed + $2500 sale now) Court holds that nonrecourse and recourse debt are to be treated alike: o The loan is to be included in the basis of the asset it finances o When the property is sold, the balance remaining on the loan is included in the AR Result:

Tax Outline Warren Spring 2007

91 of 104

o This helps the taxpayer because he can now finance asset acquisition with loans and then take depreciation deductions based on a basis that includes those loans You add loan to basis (and are eligible for depreciation deductions) no matter who put the money in Franklin Purchase Money Mortgage o Buyer put a down payment on property ($75k), o Leases it back to the seller, then rents it for $9k/yr for 10 yrs o Then buyer promises to pay the balance ($975k) at a specified time in the future [Nonrecourse loan from seller to buyer] Buyer wants to start taking depreciation deductions on the FULL purchase price o Commissioner concerned about taxpayer inflating the purchase price (which he has no real intention of paying) in order to claim depreciation deductions in the interim On one hand, we have the buyer realizing the full basis up front and taking depreciation deductions from it immediately On the other hand, the seller is deferring his income via the installment method o Cant say youre putting more in than the asset is actually worth Court rules that purchase price was far more than FMV taxpayer gets no basis in the property for depreciation purposes (less danger of this same artificial inflation when theres a 3rd-party lender because the lender will want loan backed up with asset of equivalent value) Italian Movie Hypo Facts o Buy rights Italian movie and promise to pay $1M in 10 yrs o You depreciate the movie (before its even finished) over the first 3 years and take deductions against your salary ($333k, $333k, $333k) o The movie dies and you cant pay the man; he cancels your debt, so technically youre forced to include the $1M under Cancellation of Indebtedness, thereby cancelling out what you were able to deduct in depreciation BUT you benefited from being able to take those depreciation deductions early on incentive to make the movie cost as much as possible up front Analysis: o Technically the govt won in Crane here by making the law reflect the full earnings from borrowing BUT they opened the door for all of this depreciation deduction abuse like we see in Franklin and this hypo

Tax Outline Warren Spring 2007

92 of 104

Saying the thing is worth much more than it is and taking depreciation deductions based on that

Tufts Simplified Factual Pattern o Facts Borrows $110k (non-recourse) to buy a building for $110k Basis under Crane = $110k (though obviously you put in none of your own money) Take depreciation deductions of $70k Repay $30k of the loan ($80k outstanding) Property FALLS in value to $20k at which pt: surrendered to lender o Real-World: Borrower lost $30k (loan repayment) Lender lost $60k (put out $110, but got back $30 from loan + $20 property) o How to deal with taxable income? Taxpayer He says his AR was $20k (most he could get if he sold it) o Quotes fn37 from Crane: if the value of the property is LESS than the mortgage, a borrower who is not personally liable cannot realize a benefit equal to the mortgage (saying that if the debt is more than the FMV of the property, AR cant be the higher debt value) o Since its nonrecourse debt, hes effectively only liable for whatever the property is worth, so by cancelling this debt, hes benefiting $20k He says his basis was $40k o $110k to begin with, but hes taken $70k in depreciation deduction SO his calculation is a $20k LOSS (+ hes previously taken depreciation deduction of $70k) Overall $90k down in Taxable Income o (Hes accounted for both his own and the banks realworld loss) Majoritys Calculation SCOTUS doesnt buy Tufts calculation of AR o Forget economic benefit we put the loan in the basis, so we need to put it in the AR o Even where FMV < Outstanding Loan value, then well calculate your AR as value of the loan (Crane) AR = $80k

Tax Outline Warren Spring 2007

93 of 104

Court agrees that Basis is $40k SO now he had a $40k GAIN (AR $80k minus B $40k) + previous depreciation deduction of $70k = overall decrease in TI of $30k o [Reflects the Real World] OConnor Concurrence (Professor Barnett) OConnor would say, there really is a separate lending/borrowing that is someone cancels it, we should tax separately Splits up the transaction between Asset Disposed of and Liability Closed o Asset Disposed of AR = 20 (FMV) Basis = 40 (Original minus depreciation) (Loss of 20) o Liability Closed AR = 80 (110 loan 30 repaid) Amt Paid = 20 (FMV of property surrendered) Gain of 60 This reaches the same result as the majority (an overall gain of 40 but minus the 70 he already took) BUT there may be different tax consequences for the different types o Asset low Cap Gains rate o Liability higher Ordinary Income rate (The majoritys rule holds for nonrecourse debt, but OConnors rule has held for recourse debt under Reg. 1.1001-2(c)) Real World Tufts o Facts Taxpayers borrowed (nonrecourse) $1,851,500 + used $44,212 of their own money to construct an apartment complex ($1,895,712) Take depreciation deductions in amount of $439,972 Transfer property (with then FMV of $1,400,000) to unrelated 3rd party who assumed the nonrecourse mortgage o Real-World Borrower (Taxpayer) lost $44,212 He lost the $44,212 that he put in Lender lost $451,500 Loaned out $1,851,500 and property now worth $1,400,000 o Total loss is $495,712 o Dealing with taxable income: Taxpayer Deductible LOSS of $55,740 (use Crane) o AR = 1,400,000

Tax Outline Warren Spring 2007

94 of 104

(FMV of property surrendered lesser value!) o Basis = 1,455,740 ($1,895,712 439,972) (what the bank put in + what they put in depreciation) - previous depreciation deductions of $439,972 o TOTAL = $495,712 (Trying to deduct their real-world loss + lenders real-world loss!)

Court Taxable GAIN of $395,760 o AR = 1,851,500 (Even where FMV < Outstanding Loan value, then well calculate your AR as value of the loan (Crane)) Outstanding Loan Value = $1,851,500 o Basis = 1,455,740 (same as in taxpayers calculation) (what the bank put in + what they put in depreciation) - previous depreciation deductions of $439,972 o TOTAL Loss = $44,212 (This isolates the taxpayers real-world loss) OConnor Bifurcation She would bifurcate the $395,760 o Asset Disposed of AR = 1,400,000 (FMV) Basis = 1,455,740 (Original minus depreciation) (Loss of 55,740) o Liability Closed AR = 1,851,500 (loan not repaid at all) Amt Paid = 1,400,000 (FMV of property surrendered) Gain of $451,500 This reaches the same results (Overall Gain of $395,760) o (Then subtract the $439,972 from depreciation) o Gets to isolated Taxpayer loss of $44,212 BUT dont forget the 2 different levels of taxation for Cap Gains vs. Ordinary Income Overall lesson from Tufts o Even where FMV of the property sold or surrendered < Outstanding Loan value, then well calculate your AR as value of the loan and not of the property (Crane)

Tax Outline Warren Spring 2007

95 of 104

We cancelled your debt and while you were only liable anyway for the value of the property (since its nonrecourse debt), we include in the AR the whole value of the loan (minus whatever youve repaid) Tufts vs. Franklin o Franklin Mortgage not includible in basis (and not in AR) where amount of mortgage exceeds FMV of the property securing it Cant inflate FMV for the sake of taking a large mortgage and then taking depreciation deductions off of that (So we disallow the big mortgage in the basis and subsequently disallow it in AR) o Tufts Taxpayer must treat the nonrecourse mortgage consistently when accounting for basis and AR Well include the debt in the basis and likewise include it in AR upon foreclosure (to the extent it hasnt been repaid), even if the amount of debt exceeds the FMV of the property

DEFERRED COMPENSATION
Mechanics Part to Salary Part to pension plan that gains interest BUT is only taxed @ time of use Fair? H-S dumb (net worth rising in the interim) Consumption Tax right (youre only better off in the long term) Rationale: PATERNALISM Encourage savings o (rules say you cant limit to highly compensated employees) Cant use until later anyway Two types IRA (Deferral) o Not taxed now o Put into account and tax later on withdrawal Roth IRA (Exemption) o Tax now at time of earning o Exempt the interest later o Limits

Tax Outline Warren Spring 2007

96 of 104

$5000 salary (which is a higher limit than the regular because thats post-tax dollars) Rationale: Congress trying to collecting the money now so it encourages the Roth (kills our kids)

Tax Outline Warren Spring 2007

97 of 104

CAPITAL GAINS
Capital Gains Rationale: Investment Incentive Encourage Realization o Lock-in Effect dont want people to have to wait until death and 1014 step-up in basis Bunching o Realization forces us to realize all gain in 1 yr Lesser rationales: o Double taxation of corporate stock o Incentives to take risks o Offset taxation of gain just due to inflation Goal: o We want to connect our rationales to whatever definitions of capital asset that we use 1222 Netting LTCG = gain from sale or exchange of capital asset held for more than 1 yr o sale or exchange narrow; only those transactions that encourage realization Net Long-term and Short-term CG separately then combine them and figure out some preferences 1222(11) o Net Capital Gain = Excess of NLTCG STCL STCL = loss from assets less than 1 yr NLTCG = LTCG LTCL o (Under 11, all of your losses can be used can always deduct losses against gains) BUT if you had GAIN on Short-Term, wed tax NCG at the Long-term Gain and tax the Short-term gain as ordinary income 1(h) Rate 15% if over 1 yr and in higher bracket 5% if over 1 yr and would otherwise be taxed at 10 or 15% Mechanics o Single person with Taxable Income of $35k including $6k NCG 1st 7550 10% / Up to 30650 15% / Above 30650 25% o $29k in salary taxed as ordinary income o 1st bit of CG (from $29k up to 30650) taxed at 5% o Rest of CG taxed at 15% - (because would have been taxed at 25%) Single person is taxed at 5% for any CG income that keeps total income below $30,650, but once you hit that amount, youre taxed at 15% o We stack the ordinary income at the bottom

Tax Outline Warren Spring 2007

98 of 104

o Capital Gains at the top 1211 Capital Loss You can use losses to offset ANY capital gain (long-term or short-term) If you have net capital loss, you can only offset up to $3k/yr but you can carryover Rationale: o Prevent Cherry-picking Want to limit loss for assets that you can easily cherry pick away Net worth may not have changed, but you could realize a loss selectively in a high-income year to try and deduct 165(g) Worthless Securities If any security that is a capital asset becomes worthless, the loss shall be treated as one that is from sale or exchange o (Congress wants to bring within capital gains land so lies and says sale or exchange is met) 1222(3) 1 yr requirement Rationale: o Investment incentive want to keep money in o BUT might go against the lock-in effect 1031 Like-Kind Exchange If you exchange a like-kind asset and then sell your new asset 6 months later, you still qualify because youve been invested for over a year What is a capital asset? 1221(a)(1) Used in Business Property held by the taxpayer (whether or not connected with his trade or business), but does NOT include: o Stock in trade of the taxpayer which is inventory in business for sale to customer in the ordinary course of business If regular ordinary income If rarely buy and sell capital gain o Rationale: Investment incentive (you dont have be incentivized to invest in and carry on your own business) Types: o Buy and sell real estate daily No CG Ordinary income Rarely CG [challenge to decide this daily/rarely line] o Buy and sell art

Tax Outline Warren Spring 2007

99 of 104

If its ordinary sale to customers as part of regular/daily business ordinary income o Stocktraders OK if they buy/sell daily because theyre not selling to customers! Rationale: We sort of backed into this in the Depression because we wanted traders to have to deduct their stock losses as capital losses and not as deductions from their ordinary income Corn Products o Company locks in price of corn to protect themselves from major price fluctuation o When corn price goes up and they are able to lock in lower price, they try to claim that the profit from the futures contract is a Capital Gain Ordinary business is Corn Products; transaction was stock trading o Court says they must take it as Ordinary Income Purpose of CG was to promote realization and this type of transaction doesnt fall within that reasoning Narrowly apply CG (1221) and broadly construe all exceptions to CG (1221(a)(1) and (2)) On taxpayers theory, if the price of corn goes up, theyll realize the contract and say its CG; if the price goes down, they wont sell the contract, theyll take delivery of the corn and cost will go into ordinary income Trying to CG for Gains, but OI for losses This was not a venture within your business; youre not a legitimate capitalist o Corn future is enough like inventory under 1221(a) (1) that well keep out of CG o Results: BAD if you have gains (preferred preferential CG rate) GOOD if you have losses (able to characterize loss as OI) So youll say that losses are result of some business connection and therefore entitled to Ordinary Income deduction Arkansas Best o Response to the incentive we create in Corn Products to define losses closely related to business as ordinary losses rather than capital losses o Refuses to read motive test into 1221 Taxpayer sought to claim ordinary loss deduction on sale of stock it held for business purposes; court notes that stock is naturally viewed to a capital asset, so the IS would be hard-pressed to challenge Arkansas Best if they had claimed CG on the sale of stock rather than capital loss

Tax Outline Warren Spring 2007

100 of 104

1221(a)(2) Property Subject to Depreciation Not CG under 1221 BUT 1231 allows Tax Heaven! o If you have net gains, tax as CG o If you have net losses, tax as OI We give similar treatment to Involuntary Conversions under 1231(a)(3) What counts: o Only depreciable property o NOT inventory barrier around 1221(a)(1) o (some silly selective stuff like x-mas trees) Mechanics: o Buy machine for $100k o Deduct $70k depreciation o Sell machine for $100k Basis = $100k Adj Basis = $30k o How to treat the $70k Gain? Depreciation Deduction was from OI But maybe the current gain can be CG?!?! NO Recapture o 1245 Machines o 1250 Real Estate You must recapture the depreciation deduction as OI (anything you make beyond the depreciation deductions are OK in the CG World o Advantage: Tax deferral (able to take deductions early on) Maybe you have lower OI rate @ time 2, so you benefit 1221(a)(3) Income from Personal Services A copyright, literary/musical/art composition, letter or memo held by the taxpayer whose personal efforts created it is not a CG o Exs. Painter sells painting sure its property, but well do OI Give to daughter and she sells Not taxable to her when she gets it (carryover basis) BUT 1221(3)(c) if her basis is determined by gift from artist, then she gets OI too Art Collector gets CG (not business) o Rationale: Encourage realization (artist will sell it anyway

Tax Outline Warren Spring 2007

101 of 104

Charitable contributions from artist: o FMV = 105, Basis = 5 o 170(e)(1)(a) Amount of any charitable contribution of property shall be reduced by amount of gain which would not have been long-term CG if sole by TP and FMV If sale would not have been CG, then well reduce deduction to basis in asset If sale would have been CG, you can deduct up to FMV Ex. Artist gave property worth 105, but can only deduct 5 (no double benefit) If collector gives painting worth $2 zillion, he is entitled to $2 zillion deduction (double benefit) 1221(b)(3) Country music exception o If youre not an artist, but rather you sell country music songs as a writer, well allow that to be CG (treat normal daily job as CG) Note that the charity exception in 170(e)(1)(a) denies this

1235 Patents These counts as capital assets Rationale: o Encourage innovation o Want people to invest in patents Sale of Going Business Run a hardware store: o Nails, Fixtures, Building & Real Estate, Other Assets When you sell, need to figure out what part of sale price is allocated to each asset o Nails inventory (OI) o Fixtures / Building Depreciable Property subject to 1231 + recapture but might be CG o Other asset (goodwill) 1221 CG Substitutes for OI Our usual line is: o Recurring Receipts OI Rent Interest on bonds Dividends o Underlying Property CG Real Estate Bond Stock

Tax Outline Warren Spring 2007

102 of 104

Hort Taxpayer is lessor letting someone out of lease o AR $140k, but reports LOSS of $21,494.75 (Basis would have to be $161,494.75) Basis should be value of that lease and cancellation was $21k less than that His argument: o My asset = right to receive above-market rent (premium lease) o Thats disappearing, so I should get paid Cancelled out/sold asset CG SCOTUS: o The $140,000 is a substitute for recurring receipts (OI) o Cancellation is merely a substitute for rent reserved in the lease Advance payment of recurring receipt (TP has not sold the underlying land) McAllister Sale of life interest in a trust o Selling income interest (anticipating rent) o Payment up front in value of future income Money received in exchange for giving up life interest in a trust is CG rather than advance payment of OI Blair analogy: o Court concludes that interest in the trust was a PROPERTY interest, not just income Distinctions from Horst: o Substantiality of interest conveyed (gave away the tree) o Equitable nature of the interest o Complete divestiture of interest (not just a carve out) Gift Issue Income Donor/Donee taxed on later income Sale OI or CG to seller

Horst (gift is income) Hort (lease cancellation) Donor is taxed (he Taxed as OI (sale of kept he tree) income right) PG Lake

Property

Blair (life interest) Donee taxed

McAllister- (life interest) CG

Tax Outline Warren Spring 2007

103 of 104

(recipient)

Commissioner v. PG Lake Company cancels debt to President and gives him oil payment in the amount of $600k ORDINARY INCOME Court says form will not control o TP has enjoyed fruit of his labor Purpose of CG was to relieve tax burden on gains resulting from a conversion of capital investments o Remove deterrent effect of realization Ferrer Taxpayer sold rights under a Dramatic Production contract o Right to lease the play o Right to prevent the movie o 40% share of movie profit Friendly analyzes whether each part separately would be enforceable in court to get injunction o Lease of the play injunctive relief o Power to Prevent movie injunctive relief Therefore, these are Capital Assets o 40% share of the movie no equitable relief Therefore, this is Ordinary Income CBS Rental Hypo CBS leases TV studio for 60 yrs with obligation to restore property to original condition at the end of the term When CBS renegotiates, the leasor reminds them of the original obligation CBS buys them out o How to characterize this payment: Ferrer would make it capital gain equitable interest The tax code would make it capital gain its a replacement for seats/other depreciable stuff in the studio Hort would say its just a substitute for rent (ordinary income) you could have charged more rent and not had this obligation Laterra Taxpayer won lottery for $10M Enter into sale where he sells right to receive Govt cites Hort substitute-for-ordinary-income test 3rd Circuit says that in theory, all capital assets are substitutes for ordinary income o 3 Part Test:

Tax Outline Warren Spring 2007

104 of 104

General type of income that taxpayer is taking in Family Resemblance we knew things that are definitely CG and definitely OI (which of these 2 things is this most like?) Carve Out Have you given up all of your property? o Blair / Horst / McAllister / Hort They talk about 2 kinds of carveouts: o Horizontal retain interest in your property (Horst) o Vertical give out just part of the principal This is a vertical carve-out because the remainder was retained by the taxpayer he kept part of the tree Earned vs. Unearned Income Earned already been earned just have to wait for it Unearned must do more for it o LaTerra was earned because all he had to do was to wait for the lottery receipts to roll in OI

GOAL for CAP GAINS Try to figure out which rationales/tests make the most sense Code tries to fit one property definition for both: o lower tax rates on certain gains o Preventing cherry-picking of certain losses

Do we really have a Realization Requirement? One extreme: Eisner o System says realization is the only way to go because doing change in value is too complicated Middle ground o OID (tax you on interest in interim: 10 + 11 + 12) o Constructive Realization (havent closed transaction, but have given up on gain or loss) Other extreme: Haig-Simons (changes in value) o Mark to Market (1256) taxes on changes in value for sophisticate futures dealers

Anda mungkin juga menyukai